Download as doc, pdf, or txt
Download as doc, pdf, or txt
You are on page 1of 47

1

CHOLINOMIMETICS. CHOLINESTERASE 6. A doctor administered injection of galanthamine to a 63 E. Atropine


INHIBITORS years old patient after ischemic insult of the brain for 12. Drugs from this group are used to decrease secretion of
recovery of functions of the CNS. What is the mechanism salivary and gastric glands, eliminate bronchospasm and
1. In clinical practice quite often there are cases of of action of this drug? bradycardia. Indicate the group of drugs.
poisoning by phosphororganic substances (insecticides, A. *Inhibition of acetylcholinesterase A. *M-cholinolytics
pest-Killers). Alloxim is the drug used to treat this B. Inhibition of cholinacetylase B. Myorelaxation drugs
poisoning. Specify the group of drugs to which it belongs. C. Inhibition of catechol-O-methyltransferase C. M-cholinomimetics
A. * Regenerators of cholinesterase D. Inhibition of dopamin-beta-hydroxylase D. Cholinesterase inhibitors
B. M-cholinoblockers E. Inhibition of monoamine oxidase E. Cholinesterase regenerators
C. Sympathomimetics 7. A patient was paralyzed after insult. Indicate the drug 13. An 8 years old child was poisoned by mushroom fly-
D. Adrenomimetics which can be administered to him for recovery of agaric. Which of the following drugs should be used as an
E. N-cholinoblockers movement function in paralyzed extremities? antagonist?
2. A patient with the diagnosis of glaucoma received A. *Galanthamine A. *Atropine
proserinum (neostigmine) in the form of eye drops. What B. Aceclidine B. Pirenzepine
compound is inactivated by proserinum that causes the C. Atropine C. Morphine
decrease of intraocular pressure? D. Carbacholine D. Ipratropium bromide
A. *Acetylcholinesterase E. Mellictinum E. Aceclidine
В. Butyrilcholinesterase 8. Indicate the agents used for treatment of the poisoning 14. A 40 years old man was admitted to the toxicological
C. Cholinacetyltranspherase by phosphor-organic substances? department with poisoning by insectiside from the group of
D. Pseudocholinesterase A. *Cholinesterase regenerators organophosphorous compounds. Which agent blocking
E. Acetylcholine B. Sympatholytics peripheral M-cholinoceptors is the most effective for the
3. Proserinum (neostigmine) was introduced to the patient C. Adrenomimetics treatment of the poisoning?
with overdosage of tubocurarine. Due to what mechanism D. M-cholinoblockers A. *Atropine
of action is proserinum effective in this situation? E. N-cholinoblockers B. Pirenzepine
A. *Inhibition of cholinesterase activity 9. A doctor administered Pilocarpine to the patient with C. Plathyphylline
B. Blockade to the presinaptic membrane glaucoma. What is the main effect of this agent? D. Benzohexonium (hexamethonium)
C. Activation of M-cholinoceptors A. *Decrease of intraocular pressure E. Amizylum (benactlzine)
D. The increase of cholinesterase concentration B. Increase of the cardiac rhythm 15. Alloxim is used for treatment of poisonings with
E. Blockade of adrenoceptors C. Stimulation of GIT peristalsis phospho-organic insectiscides and strong choline esterase
4. What drug is used in intestinal atony? D. Increase of salivation inhibitors. Indicate its mechanism of action.
A. *Proserinum (neostigmine) E. Increase of myometrium contructility A. *Regeneration of cholinesterase.
B. Benzohexonium (hexomethonium) 10. A patient with complains of dryness of the oral cavity B. Blockade of n-cholinoceptors.
C. No-spa (drotaverine) visited a dentist, who made the diagnosis: xerostomia. C. Stimulation of noradrenaline release
D. Atropine Which of the following drugs should the dentist prescribe? D. Excitation of adrenoceptors.
E. Pirilenum fpempidinej A. *Pilocarpine E. Blockade of m-cholinoceptors.
5. A 5 years old boy with the diagnosis suffers from B. Atropine 16. Patient with complaints of dryness in the mouth,
disorders of movements coordination and muscular C. Methacinurn photophobia and vision violation was admitted to the
weakness (predominantly in the right leg) after D. Ipratropium bromide reception-room. Skin is hyperemic, dry, pupils are dilated,
poliomyelitis. What drug should be administered to E. Halazolinum (xylomethazoline) tachycardia. Poisoning with belladonna alkaloids was
improve neuromuscular transmission? 11. A dentist prescribed an agent stimulating salivation to diagnosed on further examination. What medicine
A. *Proserinum (neostigmine) a patient with xerostomia. Indicate the drug. should be prescribed?
B. Coffeinum A *Aceclidine A *Prozerin
C. Phenaminum (amphetamine) B. Dithylinum (suxamethonium) B Diazepam
D. Extractus Eleutherococci C. Armin C Pilocarpine
E. Aethimizolum D. Scopolamine D Armine
2
E Dipyroxim C. Morphine D. Astringent drugs
17. Analeptical remedy of reflective type from the H- D. Omnoponum E. M-cholinomimetics
cholinomimetics group was given to the patient for E. Promedolum 7. A 6 years old child was delivered to the hospital with
restoration of breathing after poisoning with carbon 2. Indicate mechanism of broncholytic action of following symptoms: motor and psychical excitement, dry,
monoxide. What medicine was prescribed to the patient? metacinium. hot and hyperemic skin, hyposalivation, difficulty of
A *Lobeline hydrochloride A. *Blockade of m-cholinoreceptors of bronchi. swallowing and hoarse voice, dilated pupils and
B Atropine sulphate B. Stimulation of m-cholinoreceptors of bronchi. photophobia and tachycardia. From the anamnesis it is
C Adrenalin hydrochloride C. Stimulation of beta-2~adrenoreceptors of bronchi. known that the child has eaten some berries with dark-
D Mesaton D. Blockade beta-2-adrenoceptors of bronchi. violet colour. Indicate an alkaloid which caused this
E Pentamin E. Myotropic spasmolytic action. poisoning
18. A patient suffering from myasthenia has been 3. A 50-year-old male farm worker was admitted to the A. *Atropine
administered proserin. After its administration the patient emergency room. He was found fainted in the orchard and B. Pirenzepine
has got nausea, diarrhea, twitch of tongue and skeletal since then has remained unconscious. His heart rate is 45 C. Ipratropium bromide
muscles. What drug would help to eliminate the and his blood pressure is 80l40 mmHg. He is sweating and D. Plathyphylline
intoxication? salivating profusely. Which drug from the following E. Methacinum
A *Atropine sulfate should be prescribed? 8. In order to do eye inspection, it is necessary to widen the
B Physostigmine A. *Atropine pupils. Choose the agent which can be used for this
C Pyridostigmine bromide B. Physostigmine purpose.
D Isadrine C. Proserine A. *Atropine
E Mesatonum D. Pentamine B. Amizylum (benactizine)
19. A patient in postoperative period was prescribed an E. Norepinephrine C. Pilocarpine
anticholinesterase drug for stimulation of intestinal 4. The patient was admitted to a hospital with following D. Noradrenaline
peristalsis and tonus of urinary bladder. What drug is it? symptoms: general excitement, dry and hyperemic skin, E. Acetylcholine
A *Proserin dryness of the oral cavity, disorder of vision, dilated pupils 9. Pharmacological effects of this drug substance are
B Dichlothiazide and photophobia, tachycardia. The doctor made the midriasis, decrease of exocrine glands secretion,
C Reserpine diagnosis: the poisining by belladonna's alkaloids. Indicate tachycardia, dilation of the bronchi, inhibition of intestinal
D Mannitol the main alkaloid of this plant? peristalsis. This drug does not penetrate into the CNS.
E Propanolol A. *Atropine Determine the drug.
20. On the 2-3rd day after stomach resection intestinal B. Aceclidine A. *Methacinum
peristalsis wasn't restored. What is to be administered for C. Pilocarpine B. Atropine
stimulation of gastrointestinal tract? D. Armin C. Adrenaline
A *Proserin E. Galanthamine D. Isadrinum (isoprenalinej
B Prasosin 5. A patient suffering from bronchial asthma has E. Pirenzepine
C Cyclodole accompanying disease glaucoma. Indicate the group of 10. Atropine sulfate was administered to the patient for
D Atropine sulfate drugs which is contraindicated for the patient. treatment of intestinal colic. What accompanying disease
E Acetylcholine A. *M-cholinotytics confines usage of the drug?
В. Myotropic broncholytics A. *Glaucoma
M-CHOLINOBLOCKERS C. Alfa-beta-adrenomimetics B. Bronchial asthma
D. Glucocorticoids C. Sinus bradycardia
1. A 48 year-old man had been admitted to the urology E. Beta-2-adrenomimetics D. Hypotension
department with signs of renal colic. Indicate the drug 6. A dentist used a drug to inhibit salivation in a patient E. Dizziness
which main effect is associated with relaxation of smooth during treatment. Indicate the group this drug belong to. 11. A 48-years-old man was admitted to the urologic
muscles A. *M-cholinolytics department with symptoms of renal colic. What drug from
A.* Platyphylllnum B. Beta-adrenoblockers mentioned below can be used for smooth muscles
B. Analginum C. Beta-adrenomimetics relaxation due to blockade of M-cholinoceptors?
3
A. * Plathyphyllin B. Strophanthinum B. Withdrawal syndrome
B. Omnopone C. Bemegridum C. Inhibition of the CMC
C. Morphin D. Cordiaminum D. Disorder of gustatory sensibility
D. Anaiginum E. Ethyl alcohol E. Diarrhea
E. Promedol 2. During operation on the thyroid gland, to prevent 7. What neurotropic hypotensive agent belongs to the
12. The agent inhibiting vestibular centers is used for sea excessive hemorrhage the doctor decided to use a method group of ganglion blockers and is used to eliminate
sickness treatment. Determine this drug. of controlled hypotension with the help of trickling hypertensive crisis?
A. *Scopofamine intravenous introduction of a drug. Specify it. A. *Pentaminum (azamethonium)
B. Atropine A. * Hygronium B. Octadinum (guanethidine)
C. Plathyphylline B. Pirilenum C. Anaprilinum lpropranolol)
D. Methacinum C. Pentaminum D. Dopamine
E. Homatropine D. Pachycarpinum E. Reserpine
13. A stomatologist injected a patient with a certain drug in E. Dimecolmum 8. 0,1% solution of hygronium was introduced intrave-
order to reduce salivation during tooth filling. What drug is 3. Injection of dithylinum (which had been introduced for nously in drops to a 50-years-old patient with increased BP
it? simplification of reposition of a dislocation in a shoulder (220l110 mmHg). What is the mechanism of action of the
A *Atropine sulfate joint) evoked apnea in the patient. What is it necessary to drug?
B Adrenaline hydrochloride introduce to the patient for restoration of breathing? A. *Blockade of N-cholinoceptors
C Proserin A. * Fresh citrated blood B. Blockade of M-cholinoceptors
D Pilocarpine hydrochloride B. Bemegridum C. Blockade of adrenoceptors
E Mesaton C.Dipiroximum D. Blockade of calcium channels
14. A patient with drug intoxication presented with the D. Isonitrosinum E. Stimulation of alfa-adrenoceptors
dryness of oral mucous membrane and mydriatic pupils. E Galanthaminum 9. Sings of tubocurarine overdosage appeared in a patient
Such action of this drug is associated with the following 4. A 53 year old man was admitted to a hospital in severe during operation. What drug should be used as an
effect: state with complaints of headache, vertigo, nausea. BP antagonist?
A *Muscarinic cholinoreceptor block 220/120 mm Hg. After injection of 1ml of 2,5% A. *Cholinesterase inhibitors
B Muscarinic cholinoreceptor stumulation benzohexonium solution the patient's state improved. B. Alfa-adrenomimetics
C Nicotinic cholinoreceptor stumulation Indicate the mechanism of action of this agent. C. M-cholinoblockers
D Adrenoreceptor stimulation A. *Blockade of N-cholinoceptors of vegetative D. Ganglion blockers
E Adrenoreceptor block ganglions E. beta-adrenomimetics
15. Introduction of a pharmaceutical substance to an B. Blockade or M-cholinoceptors 10. A 45-year s-old man with dislocation of shoulder joint
experimental animal resulted in reduction of salivation, C. Blockade of beta1-adrenoceptors was admitted to the hospital. What drug can be used to
pupil mydriasis. Next intravenous introduction of D. Excitation of alpha- adrenoceptors relax skeletal muscles and set the bone?
acetylcholine didn't lead to any significant changes of heart E. Blockade of alpha1-adrenoceptors A. *Dithylinum (suxamethonium)
rate. Name this substance: 5. An agent from the group of ganglion blockers was B. Dimedrolum (diphenhydramine)
A *Atropine administered to a patient with essential arterial hyper- C. Analginum (methamizole)
B Adrenaline tension. What effect underlies the decrease of BP? D. Promedolum (trimeperidine)
C Propranolol A. * Sympathetic ganglions blockade E. Acetylsalicylic acid
D Proserin B. Blockade of adrenal cortex 11. Peripheral myorelaxant was introduced to a patient
E Salbutamol C. Blockade of carotide sinuses with fracture of humeral bone to facilitate the bona
D. Vasomotor centre blockade reposition. Respiratory standstill developed in the patient.
N-CHOLINOBLOCKERS E. Parasympathetic ganglions blockade The respiration restored after introduction of fresh citrate
6. Ganglion blocker benzohexonium (hexamethonium) was blood. What myorelaxant was introduced to the patient?
1. Indicate the drug used for the treatment of pulmonary introduced to a patient with hypertensive crisis. What A. *Dithylinum (suxamethonium)
edema caused by systemic arterial hypertension complication can develop in the patient after introduction? B. Tubocurarine
A. * Benzohexonium A. *Orthostatic hypotension C. Pancuronium
4
D. Pipecuronium B Catalase C. Eohedrine
E. Vecuronium C Acetylcholinesterase D. Adrenaline
12. Myorelaxant dithylinum (suxamethonium) was D Glucose 6-phosphatase E. Isadrinum
introduced to a patient with fracture of humeral bone to E Glutathione peroxidase 2. Expressed arterial hypotension had developed in the
facilitate the bone reposition. Respiratory arrest developed 17. A patient had to go through an operation. Doctors patient during an operation which had been carried out
in the patient. Proserinum (neostigmine) was introduced to introduced him dithylinum (listenone) and performed under phthorotanum-general anesthesia. Which- from the
a patient (it was the doctor's mistake), but respiration intubation. After the end of operation and cessation of listed medicines below should be introduced to the patient
didnTt restore. What drug can be used? anesthesia the independent respiration wasn't restored. to normalize he's arterial blood pressure?
A. *Fresh citrate blood Which enzyme deficit prolongs the action of A. *Mesatonum
B. Dipyroxime muscle relaxant? B. Strophanthin
C. Isonitrosine A *Pseudocholinesterase C. Ephedrine hydrochloride
D. Galanthamine B Succinate dehydrogenase D. Noradrenatlnum hydrotartrate
E. Bemegride C Carbanhydrase E. Adrenaline
13. Dithylinum (suxamethonium) was introduced to a D N-acetyltransferase 3. Indicate mechanism of broncho-lytic action of
patient with the aim to relax skeletal muscles during E K-Na-adenosine triphosphatase salbutamol?
operation. It led to myorelaxation during 6 hours instead of 18. A patient with fracture of his lower jaw was admitted A. *Stimulation of beta-2-adrenoceptors
5-7 minutes. This situation can develop due to genetic to the maxillofacial department. It was decided to fix his B. Inhibition of phosphodiesterase
deficiency of: bones surgically under anaesthetic. After intravenous C. Activation of noradrenaline synthesis
A. *Blood plasma cholinestherase introduction of muscle relaxant there arose short fibrillar D. Blockade of H-f-histamine receptors
B. Acetylation contractions of the patient's facial muscles. What E. Blockade of M-cholinoceptors
C. Oxidative processes muscle relaxant was applied? 4. A 40 year old patient has been suffering from bronchial
D. Methylation A *Dithylinum asthma for 10 years accompanied with cardiac arrthymia
E. Carboxylation B Tubocurarin chloride (tachycardia). Indicate adrenomimetic agent which should
14. Action of what agent is significantly prolonged in C Pipecuronium bromide be administered for elimination of bronchospasm taking
patients with genetic deficiency of buthyrilcholine D Diazepam into account accompanied heart disease.
estherase? E Melictine A. *Salbutamolum
A. *Dithylinum (suxamethonium) 19. A woman was delivered to a hospital for trachea B. Adrenaline
B. Adrenaline hydrochloride intubation. What of the following drugs should be applied C. Isadrinum
C. Midantanum (amantadine) in this case? D. Orciprenalinum
D. Tubocurarine A *Dithylinum E. Ephedrinum
E. Mesatonum (phenylephrine) B Nitroglycerine 5. Salbutamol was administered to a 30 year old woman
15. A patient with a limb fracture must be administered a C Metronidazole due to danger of having miscarriage as it causes decrease
depolarizing drug from the myorelaxant group for the D Atropine sulfate of contractile ability of myometrium. Indicate mechanism
purpose of a short-time surgery. What drug is it? E Gentamycin sulfate of sympathomimetics.
A *Dithylinum relaxing action upon the uterus of this drug.
B Tubocurarine chloride A. *Stimulation of beta-2-adrenoceptors
C Cytitonum ADRENOMIMETICS B. Stimulation of aipha-2-adrenoceptors
D Atropine sulfate C. blockade of beta- f- adrenoceptors
E Pentaminum 1. The patient of 40 years suffered from bronchial asthma D. Inhibition of monoaminooxydase
16. During an operation a patient got injection of muscle for 10 years. Acompanying this disease is cardiac E. Blockade of phosphodiesterase
relaxant dithylinum. Relaxation of skeletal muscles and arrhythmia (tachycardia). Specify the drug which may be 6. A female patient was admitted to a hospital with
inhibition of respiration lasted two hours. This condition used for elimination of bronchospasm with keeping into complaints of unpleasant sensations in the heart area,
was caused by absence of the following enzyme in blood account the acompanying disease? attacks of acute weakness, sometimes loss of
serum: A. * Salbutamolum consciousness. Examination of patient revealed
A *Butyrylcholin esterase B. Orciprenaline atrioventricular blockade. Indicate the group of drugs that
5
should be appointed in this situation.n C. Bradycardia B. Methacinum
A. *beta-adrenomimetics (Isadrinum) D. Apathy C. Isadrinum (isoprenaline)
B. Cardiac glycosides (Digitoxin) E. Sleepiness D. Euphillinum (aminophilline)
C. beta-adrenoblockers (Anapritinum) 12. What drug can be used for treatment of hypotension E. Atropine
D. Calcium channel blockers (Verapamil) due to peripheral vascular insufficiency? 18. An attack of bronchial asthma developed in 40-years-
E. Sympatholytics (Ornidum) A. *alfa-adrenomimetic old woman. Indicate the drug belonging to beta2-
7. Anaphylactic shock has developed in a patient after B. b-adrenomimetic adrenomimetics which is effective for elimination of the
novocainum (procaine) injection. What agent supresses C. Analeptic attack
histamine release from mast ceils and eliminates main D. Colloidal plasma substitute A. *Fenoterol
symptoms of anaphylactic shock? E. Salt plasma substitute B. Ephedrine
A. Beclometasone 13. Mesatonum (phenylephrine) was introduced to a C. Adrenaline
B. Euphillinum (aminophilline) patient with collapse for correction of blood pressure. D. Plathvphiiline
C. *Adrenaline What is the mechanism of action of the drug? E. Atropine
D. Ketotifen A. *Stimulation of alfa-adrenoceptors 19. A 40-years-old patient has been suffering from
E. Cromolin natrium (cromoglycic acid) B. Blockade of alfa-adrenoceptors bronchial asthma for 10 years, fie has also an accompa-
8. A doctor diagnosed hypoglycemic coma in a patient C. Blockade of beta-adrenoceptors nying disease cardiac arrhythmia (tachycardia). What
with diabetes mellitus and administered glucose solution D. Stimulation of α-β adrenoceptors adrenomimetic can be administered for elimination of
IV to him. Patient's condition improved. What drug can be E. Stimulation of b-adrenoceptors bronchospasm?
used additionally as the biochemical antagonist of insulin? 14. Collapse developed in a patient due to decrease of A. *Salbutamol
A. *Adrenaline peripheral vessels tone. What preparation is the most B. Adrenaline
B. Mesatonum (phenylephrine) efficient in this situation? C. Isadrinum (isoprenaline)
C. Dobutamine A. *Mesatonum (phenylephrine) D. Atropine
D. Isadrinum (isoprenaline) B. Proserinum (neostigmine) E. Ephedrine
E. Dopamine C. Isadrinum (isoprenaline) 20. A patient with bronchial asthma has been taking
9. A patient with chronic bronchitis has been taking D. Clophelinum (clonidine) isadrinum (isoprenaline) inhalation for a long time. He
ephedrine for a long time. What is the mechanism of the E. Prazosine notices the drug leads to tachycardia and headache. Which
drug action? 15. What drugs should be used for bronchospasm agent from listed below can be used instead of isadrinum?
A. *Stimulation of noradrenaline release into treatment? A. *Salbutamol
synaptic cleft A. *beta2-adrenomimetics B. Mesatonum (phenylephrine)
B. Blockade of noradrenaline release into synaptic cleft B. Sympatolytics C. Anapriiinum (propranolol)
C. Stimulation of a-adrenoceptors C. alfa-adrenomimetics D. Dobutaminum
D. Blokade of b-adrenoceptors D. Cholines terase inhibitors E. Cordiaminum (nikethamide)
E. Direct influence on smooth muscles of bronchi E. M-cholinomimetics 21. A patient with bronchial asthma did not tell doctor that
10. Indicate the state which requires introduction of 16. Salbupart (salbutamol) was introduced to a 30-years- he had attacks of stenocardia sometimes. The doctor
ephedrine? old pregnant woman with threatened abortion. It reduced administered to him the drugs. After a patient started to
A. *Arterial hypotension contractile activity of myometrium. Indicate the take this drug, attacks of bronchial asthma became rare but
B. Caffeine poisoning mechanism of action of salbutamol? attacks of stenocardia became more frequent. Indicate the
C. Tachycardia A. *Stimulation of beta2-adrenoceptors drug which was administered by the doctor?
D. Arterial hypertension B. Blockade of beta f-adrenoceptors A. *Isadinum (isoprenaline)
E. insomnia C. Stimulation of alfa2-adrenoceptors B. Salbutamol
11. A patient with obstructive bronchitis has been taking D. Inhibition of monoaminooxidase C. Euphillinum (aminophilline)
ephedrine for a long time without doctor's control. What E. Inhibition of phosphodiesterase D. Cromolin natrium (cromoglycic acid)
side effect can be observed in the patient? 17. Indicate broncholytic drug from the group of selective E. Fenoterolum
A. *Excitation of CNS beta2-adrenomimetics. 22. A patient with bronchial asthma had been taking
B. Hypotension A. *Salbutamol tablets which caused insomnia, headache, increased blood
6
pressure. What medicine can cause such complications? D. Sympatholytics
A. *Ephedrine ADRENOBLOCKER AGENTS E. Angiotensin converting enzyme inhibitors
B. Isadrinum 6. A 40-years-old patient suffers from cardiovascular
C. Cromolin sodium 1. The patient suffering from idiopathic hypertensia, had diseases: arterial hypertension of hyperkinetic type and
D. Euphyline the acompanyng diseases: ciliary arruthmia, stenocardia, high blood renin level, stenocardia and sinus tachycardia.
E. Oxprenololum and chronic bronchitis. The physician has decided to use a Indicate the most expedient group of drugs for treatment of
23. Patient with bronchial asthma was taking tablets which drug from the group of beta-adrenoblockers. Which agent the patient?
caused insomnia, headache, increased blood pressure. should be used , taking into account the acompanyng A. *beta-adrenoblockers
What medicine can cause such complications? diseases? B. Organic nitrates
A *Ephedrine A. *Metoprololum C. alfa-adrenoblockers
B Adrenaline B. Timololum D. Sympatholytics
C Chromolin sodium C. Anaprinilum E. Ganglion blockers
D Euphyline D. Pindololum 7. Indicate the state in which nonselective beta-
E Izadrine E. Oxprenololum adrenoblockers are contraindicated?
24. A patient suffers from diabetes melitus. After the 2. A patient who had been suffering from arterial A. *Bronchial asthma
regular insulin injection his condition grew worse: there hypertension was treated with the drug which mechanism B. Thyrotoxicosis
appeared anxiety, cold sweat, tremor of limbs, general of action is connected with exhaustion of noradrenalin C. Cardiomyopathy
weakness, dizziness. What preparation can eliminate these content in sympathetic nerve endings. Indicate this drug. D. Myocardial infarction
symptoms? A. *Reserpinum E. Arterial hypertension
A *Adrenaline hydrochloride B. Clopheinum 8. Anaprilin (propranolol) was administered to a patient
B Butamide C. Anaprinilum with hypertension that normalized BP fast. What is the
C Caffeine D. Prazosine mechanism of action of this drug?
D Noradrenaline E. Dibazolum A. *Blockade of beta1- and beta2-adrenoceptors
E Glibutide 3. A 40 year old patient suffers from arterial hypertension B. Blockade of beta1- adrenoceptors
25. An ophthalmologist used a 1% mesaton solution for the with hyperkinetic type of circulation and increased level of C. Inhibition of phosphodiesterase
diagnostic purpose (pupil dilation for eye-ground renin, sternocardia, sinus tachycardia. Indicate the group D. Blockade of alfa1-adrenoceptors
examination). What is the cause of mydriasis induced by of drugs which is more preferable for treatment of this E. Stimulation of atfa2-adrenoceptors
the drug? patient. 9. Аnарrilin was administered to a patient with arterial
A *Activation of alpha-1 adrenoreceptors A. *beta-adrenoblockers hypertension accompanied by obstructive bronchitis. After
B Activation of alpha-2 adrenoreceptors B. Organic nitrates that the attack of bronchospasm occurred in the patient.
C Block of alpha-1 adrenoreceptors C. α-adrenoblockers Indicate the reason of this side-effect.
D Activation of beta-1 adrenoreceptors D. Sympatholytics A. *Blockade of beta 2-adrenoceptors of bronchi
E Activation of M-cholinoreceptors E. Ganglion blockers B. Stimulation of beta 2-adrenoceptors of bronchi
26. A patient ill with bronchial asthma didn't inform his 4. Indicate the drug which possesses hypotensive action C. Blockade of alfa 2-adrenoceptors of bronchi
doctor that he had attacks of stenocardia. Doctor exactly due to decrease of vascular tone. What drug can be D. Blockade of beta1-adrenoceptors of bronchi
administered him a medication, which taking resulted in used? E. Stimulation of alfa 1-adrenoceptors of bronchi
less frequent attacks of bronchial asthma, but A. *α-adrenoblocker 10. Therapeutic effect of beta-adrenoblocker propranolol
stenocardia attacks became more frequent. What B. N-cholinoblocker during the treatment of stenocadia is explained by:
medication was administered? C. α- β - adrenoblocker A. *Decrease of myocardium oxygen demand
A *Isadrin D. M-cholinoblocker B. Inhibition of catecholamines' production
B Salbutamol E. β-adrenoblocker C. Dilation of coronary arteries
C Aminophylline 5. Indicate the group of drugs to which prazosine belongs. D. Increase of sensibility to catecholamines
D Cromolyn sodium A. *α-adrenoblockers E. Increase of peripheral arteries resistance
E Phenotherol B. Cardioselective β-adrenoblockers 11. Examination of the 42-years-old women revealed
C. Nonselective beta-adrenoblockers stenocardia with following signs: BP = 170/100 mmHg,
7
heart rate - 84lmin, on ECG -extrasystoles. Which drug A. *Reserpin 1. A driver felt sharp pain in the eye. He was delivered to
from listed below is the most suitable for treatment? B. Anaprilin (propranolol) the hospital. What local anesthetic may be applied for
A. *Anaprilinum (propranolol) C. Pirroxanum removal of a foreign body from the eye?
B. Euphillinum (aminophilline) D. Dibazolum (bendazole) A.* Dicainum
C. Nitroglycerin E. Clophelinum B. Novocainum
D. Carbocromen 17. A 50 years old woman suffering from hypertension has C. Lidocainum
E. Papaverine taken a drug. In an hour BP was increased, but in 2 hours it D. Trimecainum
12. Beta-adrenoblocker was prescribed to a patient for the started to decrease. Indicate the drug. E. Sovcainum
treatment of ischemic heart disease but after some time A. *Octadine (guanethidine) 2. As a result of the influence of terminal anesthesia which
cough and bronchospasm occured. Indicate the agent B. Reserpine part of the skin and mucus membranes are affected.
which was administered? C. Prazosin A. * Sensory nerve endings
A. *Anaprilinum (propranolol) D. Dibazolum (bendazole) B. Epiderm
B. Talinolol E. Propranolol C. Subcutaneous fatty tissue
C. Atenolol 18. A patient with arterial hypertension has been treating D. Walls of capillaries
D. Phenigidin (nifedipine) with reserpine for a long period of time. 2-3 weeks ago he E. Dermis
E. Metoprolol began to notice stomachache, heartburn, nausea. Indicate 3. Indicate the principle of action of covering drugs.
13. A patient suffers from arterial hypertension the group of drugs which are able to eliminate these A. *Creation of protective layer on the mucous
accompanied by chronic obstructive bronchitis. Indicate symptoms? membranes.
hypotensive agent which is contraindicated for the A. *M-cholinoblockers B. Blockade of mucous membranes receptors.
patient ? B. Astringent drugs C. Coagulation of proteins of superficial layer of
A. *Anaprilinum (propranolol) C. Antacids mucous membrane.
B. Prazosine D. Proton pump inhibitors D. Formation of complexes with toxic agents.
C. Nifedipin E. H2-histaminoblockers E. Stimulation of regenerative processes.
D. Dichfothiazidum (hydrochlorthiazide) 19. Elongation of P-Q interval was revealed on ECG. 4. Indicate the mechanism of action of local anesthetics.
E. Captopril Indicate the drug which can cause this effect. A. *Block sodium channels.
14. Which of the following antiarrhythmic drug is A. *Atenolol B. Create albuminates with plasma proteins
contraindicated for the patient with cardiac arrhythmia B. Prazosin C. Block M-cholinoreceptors
accompanied by bronchial asthma? C. Reserpine D. Inhibit nonspecific activating systems of the
A. *Anaprilinum (propranolol) D. Qctadinum (guanethidine) CNS.
B. Verapamil E. Phentoiamine E. Block alpha adrenoreceptors.
C. Aimalin 20. A patient who had been suffering from severe form of 5. Why not used Novocaine is terminal
D. Nifedipin arterial hypertension after examination received diagnosis anesthesia?
E. Novocainamidum (procainamide) of pheochromocytoma (tumor of adrenal medulla which is A. *Is poorly absorbed through normal skin surface
15. Ischemic heart disease accompanied by cardiac accompanied by increased synthesis of adrenaline). and mucous membrane
arrhythmia was diagnosed in a 50 years-old patient. Indicate the group of drugs which should be administered B. Doesn't cause covering action.
Indicate the group of drugs which should be administered? to a patient before surgical treatment. C. Is rapidly absorbed and inhibits the CNS.
A. *Beta-adrenoblockers A. *alpha-adrenoblockers D. Irritates mucous membrane.
B. Alfa-adrenoblockers B. Calcium channel blockers E. Activates m-cholinoreceptors.
C. Beta-adrenomimetics C. Sympatholytics 6. Indicate main effect of the local anesthetics.
D. Alfa-adrenomimetics D. Ganglion blockers A. *Eliminate all kinds of sensibility due to blockade
E. Sympatholytics E. beta-adrenoblockers of action potential creation
16. An agent causing decrease of noradrenaline content in B. Selective relieve ot pain sensibility in local action.
vesicles of sympathetic nerve endings was administered to LOCAL ANESTETICS, ADSORBING, COVERING, C. Decrease of excitability of nerve endings
a patient suffering from arterial hypertension. Indicate this IRRITATIVE DRUGS D. Decrease of excitability and conductivity of the
drug ? 1. Agents, affecting the afferent innervation afferent
8
E. Eliminates all kinds of sensibility due to paralysis of E. Dicainum (tetracaine) C. Decrease in trypsin activity
the CNS. 13. Injection of a local anesthetic has to be given to a D. Decrease in bile secretion
7. Indicate the mechanism of action of local anesthetics. patient for tooth extraction. What drug from listed below E. Diarrhea
A. *Blockade of Na-channels is to be chosen? 19. A nurse used mustard plaster with water of more than
B. Formation of albuminates with tissue’s proteins A. *Lidocaine 60oC temperature and applied it on patient's back. In 30
C. Blockade of M-cholinoceptors B. Dicainum (tetracaine) minutes she found that patient's skin under the (sinapism,
D. Inhibition of non-specific excitatory systems of C. Anesthezinum (benzocaine) mustard poultice) mustard plaster did not get red. What is
CNS D. Cocaine the reason for absence of (sinapism, mustard poultice)
E. Blockade ot alfa-adrenoceptors E. Ketamine mustard plaster effect?
8. What morphological elements of skin and mucous 14. This agent is poorly soluble in water, so it is used for A. *Inactivation of mirosine
membranes are involved in interaction with the drug in superficial anesthesia only in the form of ointment, paste B. Inactivation of choline estherase
terminal anesthesia? and powder. What is this drug? C. Activation of mirosine
A. *Sensitive nervous endings A. *Anesthezinum (benzocaine) D. Inactivation of monoaminooxydase
B. Epidermis B. Novocainum (procaine) E. Activation of methyltranspherase
C. Fatty tissue C. Pyromecaine
D. Capillary wall D. Trimecaine
E. Derma E Sovcainum GENERAL ANESTHETICS. ETHYL ALCOHOL
9. The patient needs an operation on soft palate. What 15. What drug has to be added to lidocaine solution to 1. A woman of 22 years had been admitted to the obstetric
method of anesthesia is the most appropriate? prolong its action? clinic for delivery. Labor activity was satisfactory, but
A. *Infiltrative anesthesia A. *Adrenaline accompanied by severe pain. Choose a general anesthetic
B. Local cooling B. Coffeinum for labour pain relief, without suppressing labor activity.
C. Conductive anesthesia C. Analginum (methamizole) A.*Nitrous oxide
D. General anesthesia D. Atropine B. Ether
E. Psychotherapy E. Anaprilinum (propranolol) C. Thiopentalum-natrium
10. The patient needs Vishnevsky paranephric blockade. 16. What is the mechanism of anti-inflammatory action of D. Phthorotanum (halothane)
What concentration of novocainum (procaine) solution astringent drugs? E. Hexenalum (hexobarbital)
should to be used? A. *They form albumin film which decreases 2. In the orthopedic clinic the patient with a recent,
A. *0,25-0,5% irritation of receptors uncomplicated dislocation of the shoulder joint was
В. 1-2% B. They inhibit excitability of membrane of the nerve delivered. Choose a general anesthetic with fast, but short-
C. 2-4% fibers term action for out-patient reposition of dislocation of
D. 4-5% C. They are able to form colloid solutions shoulder joint.
E. 0.5-1% D. They block prostaglandine synthase A.*Ketamine
11. What drugs from the group of local anesthetics are not E. They inhibit phosphorylase B. Natrii oxybutyras (oxybate sodium)
used together with sulfonamides? 17. What is the mechanism of action of covering drugs? C. Ether
A. *Novocainum (procaine) A. Blockade of receptors of mucous membrane D. Phthorotanum (halothane)
B. Sovcainum B. Coagulation of proteins of superficial layer of E. Hexenalum (hexobarbital)
C. Lidocaine mucous membrane 3. For treatment of patient suffered from chronic
D. Trimecaine C. Binding to toxic substances with complexes alcoholism the doctor has administered Teturamum
E. Ultracaine formation (disulfiram) in combination with small doses of ethanol.
12. Determine the drug which is used for all type of D. *Formation of protective layer on mucous What mechanism of action has the given drug?
anesthesia. membranes A. *Oppression of activity of an enzyme
A. *Lidocaine E. Stimulation of regenrative processes acetaldehyde oxidase
B. Anesthesinum (benzocaine) 18. What is the main indication for adsorbing drugs use? B. Rising of activity of an enzyme alcohol
C. Novocainum (procaine) A. *Intoxication dehydrogenase
D. Trimecaine B. Hvpoacidic gastritis C. Blockade of dopamine receptors
9
D. Excitation of the emetic center A.*Ketamine 13. An anaesthesiologist decided to introduce solution of
E. Blockade of central cholinoceptors B. Thiopental sodium thiopental natrium intravenously to the 55-years-old
4. A sudden sharp decrease of blood pressure arose in a C. Nitrous oxide patient for general anesthesia before gastric resection.
patient who was being operated under general anesthesia. D. Diethyl ether Indicate the dangerous side effect of thiopental natrium.
The doctor introduced adrenaline, after that fibrillation of E. Sombrevin (Propanidid) A. *Breathing standstill
heart ventricles developed. Indicate the general anesthetic 9. A patient was delivered to the surgical department in B. Expressed stage of excitement
which was used and with adrenaline might cause this connection with acute appendicitis. A drug for general C. Drug tolerance
complication. anaesthesia with expressed stage of excitement was used D. Increased of blood pressure
A *Phthorotanum (halothane) by the doctor. Determine this drug. E. Drug dependence
B. Nitric oxide A. *Ether 14. A doctor administered thiopental natrium to a patient
C. Ether B. Nitrous oxide with a fracture of mandibular for analgesia during
D. Thiopentalum-natrium C. Phthorotanum (halothane) operation. What is the principal of anaesthetic action of
E. Hexenalum (hexobarbital) D. Hexenalum (hexobarbital) this drug?
5. A patient was admitted to a neurosurgical department E. Natrii oxybutyras (oxybate sodium) A. *Impairment of interneuronal impulse transmission in
with a skull trauma. Due to progressive arising of 10. General anesthesia with usage of volatile fluid was CNS
neurological signs, the diagnosis of brain edema was made to the patient during operation on the organs of B. Interaction with opioid receptors
made. Indicate general anesthetic which is also used as the abdominal cavity. Surgical anaesthesia appeared in 5 C. Blockade of peripheral receptors
drug for brain hypoxia. minutes after the beginning of inhalation with non D. Change of emotional attitude to pain
A.*Natrium oxybutyras (oxybate sodium) significant stage of excitement. Arterial hypotension and E. Inhibition of synthesis of prostaglandins
B. Ether cardiac arrhythmia developed during operation. There was 15.Thiopental natrium was introduced to the patient for the
C. Ketamine fast awakening after general anesthesia without depres- initial general anaesthesia that caused the development of
D. Propanidid sion. Indicate the drug that was used for general laryngospasm and hypersalivation. Select a drug for the
E. Phthorotanum (halothane) anesthesia. prevention of unwanted effects.
6. Indicate the general anesthetic which causes long and A. *Phthorotanum (halothane) A. *Atropine sulfate
expressed stage of excitement. В. Nitrous oxide B. Adrenaline hydrochloride
A.*Ether C. Ether C. Proserine
B. Phthorotanum (halothane) D. Natrii oxybutyras (oxybate sodium) D. Alloxim
C. Nitric oxide E. Hexenalum (hexobarbital) E. Pirroxane
D. Natrii oxybutyras (oxybate sodium) 11. Acute affection of the liver (hepatitis) appeared after 16. The patient suffers from cerebral trauma that caused
E Hexenalum (hexobarbital) the repeated introduction of this drug for general brain edema and hypoxic convulsions. Determine the drug
7. Indicate the emetic agent directly exciting vomiting anesthesia. What drug can cause this pathology? possessing anticonvulsive action.
centre which is used in treatment of chronic alcoholism. A. *Phthorotanum (halothane) A. *Natrii oxybutyras (oxybate sodium)
A.*Apomorphine B. Nitrous oxide B. Anapriline (propranololJ
B. Thermopsis grass infusion C. Thiopental natrium C. Corasole
C. Ipecacuanha root infusion D. Propanidid D. Piracetam
D. Copper sulfate E. Ether E. Cordiamintlm (nikethamide)
E. Zinc sulfate 12. Expressed bradycardia with endangered arrest of the 17. Dentist extracted the tooth in a patient under general
8. To anaesthetize the manipulation related to burn surface heart was developed during phthorotanum (halothane) anaesthesia. He used a drug with ultrashort anaesthetic
treatment, a patient was intravenously injected a short- anesthesia. What drug should be used to normalize the action. Indicate this drug.
acting general anesthetic. A minute later, the patient heart rate for continuation of the operation? A. *Ketamine
showed increase of blood pressure, tachycardia, and A. *Atropine В. Predione
increase of skeletal muscles tonicity and presence of B. Coffeinum C. Thiopental natrium
reflexes. After awakening the patient had disorientation C. Adrenaline D. Phthorotanum (haiothanej
and visual hallucinations. What medication was introduced D. Camphor E. Nitrous oxide
to the patient? E. Isadrmum (isoprenaline) 18. General anesthetic with ultra short action (5 minutes)
10
was introduced to a patient intravenously for analgesia of В. Ether C. Bromocriptin
biopsy. It caused muscular twitching, insignificant C. Chlorethyl D. Midantanum (amantadine)
decreased of blood pressure, stop of breathing for short D. Enflurane E Selegilin
period of time. Determine this drug. E Chloroform 4. A patient with convulsions was delivered by the
A. *Propanidid 24. Expressed bradycardia, reducing of blood pressure ambulance to the hospital where the diagnosis of status
В Natrii oxybutiyas (oxybate sodium) appeared during phthorotanum anesthesia. What drug is epilepticus was given. Indicate the drug of the first choice
C. Predion contraindicated to normalize the blood pressure and the in this situation.
D. Ketamlne heart rate for continuation of the operation. A. *Diazepam
E. Phthorotanum (halothane) A. *Adrenaline (epinephrine) B. Trimethinum (trimethadione)
19. Propanidid is used very often in dentistry practice. B. Atropine C. Phenobarbital
Determine the contraindication for use of this drug. C. Mesatonum (phenylephrfne) D. Dipheninum (phenytoin)
A. *Shock D. Coffeinum(caffeine) E.Carbamazepine
B. Reposition of jaw-bone fragments E. Proserine (neostigmine) 5. An attack of generalized tonoclonic convulsions
C. Reposition of dislocations of jaw accompanied by loss of consciousness and general
D. Taking out the stitches HYPNOTIC, ANTIEPILEPTIC, suppression of the CNS developed in a patient after
E. Dissection of pulp cavity ANTIPARKINSONIC AGENTS trauma. What drug should be administered to this patient?
20. The patient used solution of ethyl alcohol for warming A. *Phenobarbital
after supercooling. How does ethyl alcohol influence upon 1. The patient of 70 years has appealed to the doctor with B. Cyclodolum (trihexyphenidyl)
thermoregulation? complaints of superficial short-term sleep with often C. Levodopa
A. *Increases heat emission awakenings caused by sense of internal tension, anxiety, D. Teturam (disulflram)
В. Decreases heat emission fear. The diagnosis has been made as: senile sleeplessness. E. Midantanum (amantadine)
C. Increases heat production Make a rational choice of a hypnotic agent in the given 6. A 57-years-old woman was admitted to the hospital in
D. Decreases heat production situation. coma with inhibition of breathing, decreased BF', signs of
E. Decreases heat production and heat emission A.* Nitrazepam cardiac insufficiency, decreased body temperature,
21. Teturam (disulfiram) was administered to the patient B. Aethaminalum-natrium (pentobarbital) inhibition of reflexes. Due to anamnesis she had been
at the narcological department of psychiatric hospital. C. Phenobarbital suffering from insomnia and a doctor administered to her
Determine the indication for use of this drug. D. Bromisovalum hypnotic agent. What drug can cause this poisoning?
A. .*Chronic alcoholism E. Chloral hydrate A. *Phenobarbital
В. Acute poisoning by ethyl alcohol 2. The patient suffering from parkinsonism has been В. Sodium bromide
C. Drug addiction treating for a long time by the drug with central C. Valeriana tincture
D. Acute poisoning by morphine cholinolytic mechanism of action which efficiency has D. Scopolamine
E. Alcohol psychosis gradually decreased. Indicate drug which should be E. Promedolum (trimeperidine)
22. In what concentration is ethyl alcohol used for administered instead of used one for improving of 7. What agent should be administered to a patient in the
antimicrobial processing of skin: antiparkinsonic action? case of
A. *70% A. * Levodopa poisoning by barbiturates to normalize acid-base state?
B. 15% B. Cyclpdolum A. *NaHC03
C. 60% C. Mydocalmum B. Solution of arginine
D. 40% D. Tropacinum C. Vitamins
E. 96% E. Bellataminalum D. Antibiotics
23. This drug for general anesthesia exerts stronger action 3. A patient had been suffering from Parkinson's disease E. Physiological solution of
upon the liver and can affect the kidneys; it is used as the was admitted to the neurological department. Indicate the Natrium
main component for combined anesthesia. Phtor iones are drug inhibitting cholinergic influences which is used for 8. A 68-years-old patient was addressed to the doctor in the
formed from this drug in the process of biotransformation. treatment of this disease. polyclinic with complaints of mental disorders after usage
Determine this drug. A.*Cyclodolum (trihexyphenidyl) of Phenobarbital for a long period of time accompanied by
A. *Phthorotanum (halothane) B. Levodopa insomnia. What drug will the doctor administer to this
11
patient as hypnotic medicinal Parkinsonism. D Diazepam
A. *Nitrazepam A. *Neuroleptics E Ethosuximide
B. Cyclobarbital B. Hypnotic drugs
C. Chloral hydrate C. Antidepressants NARCOTIC ANALGESICS
D. Natrii oxybutyras (oxybate sodium) D. Tranquilizers
E. Halopendol E. Nootropic agents 1. A synthetic analgesic agent which increases contractile
9. Determine the hypnotic drug which doesn't influence 14. A patient took reserpinum for the treatment of activity of myometrium and relaxes the neck (cervix) of
the structure of sleep. hypertonic disease for a long period of time. One day the uterus was prescribed to a woman in labour. Indicate this
A. *Nitrazepam patient felt muscles weakness, restriction of motion. agent.
B. Phenobarbital Medicamentous parkinsonism was diagnosed. What drug A.* Promedolum (trimeperidine)
C. Barbital should be administered to eliminate these side effects? B. Omnoponum
D. Aethammalum-natrium (pentobarbital) A. *Cyclodolum (trihexyphenidyl) C. Analginum (methamizole)
E. Bromisoval B. Aminazinum (chlorpromazine) D. Morphine
10. The doctor administered a drug with a tranquilizing C. Haloperidolu E. Fentanylum
effect to the patient with insomnia after emotional P. Phenobarbital 2. A patient has signs of acute poisoning by morphine:
disorders. What drug was administered to the patient? E. Tubocurarine sharp miosis, loss of consciousness, decrease of the arterial
A. *Nitrazepam 15. A patient who suffers from insomnia caused by blood pressure and Cheyne-Stokes respiration. Choose the
B. Phenobarbital emotional disorder was prescribed a hypnotic drug with proper antagonist to be given.
C. Chloral hydrate tranquillizing effect. What hypnotic was prescribed? A.* Naloxonum
D. Aethammalum-natrium (pentobarbital) A *Nitrazepam B. Camphor
E. Bromisoval B Phenobarbital C. Lobeline
11. An 18-years-old patient complained of insomnia which C Chloral hydrate D. Cytitonum
manifested by not being able to sleep that led to fatigue, D Sodium ethaminal E. Cordiaminum (nikethamide)
weakness, difficulty in learning the following day. The E Bromisoval 4. A 30 years old man was admitted to a hospital due to
clinical examination revealed the following: the patient 16. A 37 year old patient suffering from obliterating fracture of the crus with dislocation and expressed pain
was easily irritated, emotionally unstable, heart rate and vascular endarteritis of lower limbs takes daily 60 syndrome. Promedolum (trimeperidine) was introduced to
arterial pressure were altered during conversations. The microgram/kilogram of phenylin. Because of presentations eliminate pain. Indicate the mechanism of action of this
doctor determined that insomnia was associated with a of convulsive disorder (craniocerebral trauma in drug.
neurosis-like state and vegetovascular distonia. Make the anamnesis) he was prescribed phenobarbital. Withholding A.* Stimulation of the opioid receptors in the
most rational choice of hypnotic drug. this drug caused nasal hemorhage. What is this CNS.
A. *Nitrazepam complication connected with? B. Blockade of the opioid receptors in the CNS.
B. Phenobarbital A *Induction of enzymes of microsomal oxidation in liver C. Stimulation of dopamine receptors in the CNS.
C. Chloral hydrate caused by phenobarbital D. Blocade of GABA receptors in the CNS.
D. Aethaminalum-natrium B Aliphatic hydroxylation of phenobarbital E. Blocade of dopamine receptors in the CNS.
(pentobarbital) C Conjugation of phenylin with glucuronic acid 5. Naloxone was introduced to a 25 years old woman
E. Bromisoval D Oxidative deamination of phenylin because of intoxication by morphine. After injection the
12. A 65-years-old woman with Parkinson's disease has E Inhibition of microsomal oxidation in liver caused
state of thebywoman became better. Indicate the
been treated with cyclodolum. Determine the mechanism phenobarbital mechanism of action of this drug.
of action of this drug. 17. A patient consulted a physician about muscle rigidity, A. *Blockade of the opioid receptors of the CNS.
A *Blockade of central cholinoreceptors constrained movements, permanent arm tremor. The B. Blockade of GABA receptors of the CNS.
B. Stimulation of dopamine receptors patient was diagnosed with Parkinson's disease. What C. Blockade of the serotonin receptors of the CNS.
C. Stimulation of serotonine receptor preparation should be administered? D. Blockade of benzodiazepin receptors of the CNS
D. Blockade of histamine receptors A *Levodopa E. Stimulation of the opiod receptors of the CNS.
E. Blockade of dopamine receptors B Phenytoin 6. A drug addict was admitted to the emergency
13. Determine the group of drug which can cause C Phenobarbital department in coma with signs of poisoning by opioid
12
analgesics. What side effect of morphine contributed to the Bone fragments reposition requires preliminary analgesia. C Promedol
poisoning? What preparation should be chosen? D Acetylsalicilic acid
A. *Tolerance A *Promedol E Butadiounum
В Euphoria B Analgin
3. Reduce of influence on which molecular substrate leads
C. Obstipation C Morphine hydrochloride
to decrease of ulcerogenic action of nonsteroid anti-
D. Inhibition of breathing D Panadol
inflammatory agents?
E. Reduction of diuresis E -
7. A 52-years-old patient who had been suffering from 19. Examination of a patient revealed extremely myotic A. *Cyclooxvgenase-1
urolithiasis was delivered to the emergency department pupils, sleepiness, infrequent Chain-Stoke's respiration, B. Kallikrein
with renal colic. A doctor administered atropine together urinary retention, slowing-down of heart rate, enhancement C. Lysosomal enzymes
with opioid analgesic with spasmolytic activity to prevent of spinal reflexes. What substance caused the poisoning? D. Cyclooxvgenase-2
development of the pain shock. Choose this drug. A *Morphine E. Adenylate cyclase
A. *Promedolum (trimeperidine) B Atropine 4. The 63 years old patient with arthritis on a background
B. Tramadol C Phosphacole of treatment by aspirin (acetyisalicylic acid) has
C. Ibuprofen D Caffeine complained of nausea, gravity in epigastrium. The doctor
D. Ketorolac E Barbital has abolished aspirin and has administered the
E. Morphine 20. A patient suffering from initial hypertension has been antiinflammatory agent from the group of selective COX-
15. Indicate the synthetic opioid analgasic which taking an antihypertensive preparation for a long time. 2 inhibitors. Indicate this drug.
possesses analgesic activity 100-400 times more than Suddenly he stopped taking this preparation. After this his A. *Meloxicam'
morphine. condition grew worse, this led to development of B. Voltarenum (diclofenac-sodium)
A.*Phentanyl hypertensive crisis. This by-effect can be classified as: С. Indomethacinum
B. Promedolum (trimeperidine) A *Abstinence syndrome D. Naproxenum
C. Pentazocin B Cumulation E. Butadionum (phenylbutazone)
D. Omnoponom C Tolerance
E. Codein D Sensibilization 5. The doctor has administered an ointment containing
16. A patient with hip fracture was prescribed a narcotic E Dependence anti-infiammatory agent from the group of pyrazolone
analgetic. Its anesthetic action is determined by interaction derivatives to the patient with arthritis of maxillofacial
with the following receptors: NON-NARCOTIC ANALGESICS joint. What agent is contained in the ointment?
A *Opiate receptors A. * Butadionum (phenylbutazone)
B Adrenoreceptors 1. Signs of gastropathy develop in the patient with B. Mefetmmic acid
C Cholinoreceptors rheumatoid arthritis who was treated with indometacin. С. Ibufrofenum
D Benzodiazepine receptors With what activity of the drug can this complication be D. Indomethacinum
E GABA-ergic receptors connected? E. Diclofenac-sodium
17. Patient in the unconscious state was admitted to the A *Anticyclooxygenase 6. Indicate the drug from the group of nonsteroid anti-
emergency room. Skin is cold, pupils are delayed, B Antiserotonin inflammatory agents which exerts the most prominent
breathing is heavy, with cycles of the Cheyne-Stokes type, C Antihistamine anti-inflammatory action in collagenoses?
blood pressure is decreased, urinary bladder is overloaded. D Antikinine A. *Iindomethacinum
Poisoning with what substance is the most likely? E Local irritating B. Aspirin (acetylsalicylic acid)
A *Narcotic analgesics 2. A female patient consulted a doctor about pain and С. Ibuprofenum
B Sedatives limited movements in the knee joints. Which of the D. Butadionum (phenylbutazone)
C Non-narcotic analgesics following nonsteroid anti-inflammatory drugs should be E. Ortophenum (diclofenac-sodium)
D М-cholinergic antagonists administered taking into consideration that the patient has
E - a history of chronic gastroduodenitis? 7. The 55 years old patient, developed gastric hemorrhage
18. A 4 year old child was admitted to the orthopaedic A *Celecoxib on the 7-th day of treatment with Indomethacinum.
department with shin fracture together with displacement. B Diclofenac sodium Indicate the pharmacological effect of the drug which
13
explains this complication. C. Bromhexinum C. Acetylsalicylic acid
A. * Diminution of formation of Prostaglandin El D. Midantanum (amantadine) D. Ibuprofenum
B. Diminution of formation of letuotrien E. Vitamin С E. Indomethacinum
С Diminution of formation of Prostaglandin E2 17. A patient with various complaints addressed to the
12/. Specify the most typical side-effects of butadionum,
D. Diminution of formation of cyclic endoperoxides doctor. After through clinical examination the following
A. *Dispeptis disorders
E. Diminution of formation of thromboxane diagnosis was made: myositis, peptic ulcer of the stomach
B. Suppression of the CNS
in remession, leukopenia. Determine the analgesic drug
8. Nonsteroid anti-inflammatory agents are effective at C. Hypothermia
for the treatment of myositis to this patient.
treatment of rheumatic disease owing to ability to oppress: D. Arterial hypotension
A. *Paracetamol (acetaminophen)
A. *Cyclooxygenase-2 E. Allergic reactions
B. Acetylsalicylic acid (aspirin)
B. Phospholipase-A2 13. The patient with systemic lupus erythematosus was C. Amitriptyline
C. Cyclooxygenase-1 treated for a long time by non-steroid anti-inflammatory D. Morphine
D. Adenylatcyclase agents. However, recently his condition has considerably E. Butadionum (phenylbutazone)
E. Peroxidase worsened. Which drug is necessary to administer to the 18. Nonopioid analgesic was administered to the patient
9. It is known, that nonsteroid anti-inflammatory drugs are patient? with neuritis of nervus trigeminus. This drug has fast
widely used for treatment of rheumatic disease. They A. * Prednisolone effect, short time of action, can cause allergic reaction of
influence what of inflammatory process? B. Analginum (Metamizole) immidiate type. It may be indicated in tablets and solution
A. *Exudative C. Dimedrolum (Diphenhydramine) for injections. Determine this drug.
B. Phase of alteration D. Thymalinum A. *Analginum (metamizole)
C. Phase of a proliferation E. Polyvitamins B. Ibuprofen
D. All phases of an inflammation 14. Non-steroidal anti-inflammatory agent was C. Mefenamic acid
E. Autoimmune administered to a patient with rheumatic polyarthritis. In D. Piroxicam
several weeks weakness and indisposition were arisen. E. Indomethacin
10. The patient was treated by nonopioid analgesic due to The clinical examination of the patient revealed necrotic 19. Pains in the small of the back developed after the
backache. In several days he started to complain of quinsy and leucopenia. What drug could cause this lifting of gravity (heavy loads). Radiculitis was diagnosed.
dyspeptic disorders and stomachache (in anamnesis the complication? It is known from the anamnesis that the patient was
patient had hyperacidic gastritis). Indicate the reason of A. * Analginum (Metamizole) suffering from ulcer of the duodenal bulb for a long period
the arisen complication. B. Paracetamolum of time. Make the most rational choice of nonopioid
A. * Inhibition of synthesis of prostaglandins in gastric C. Nitrazepam analgesics :
mucosa D. Omnopon A. *Meloxicam
B. Stimulation of parasympathetic innervation by the E. Morphine B. Ortophenum (diclofenac sodium)
stomach 15. Non-steroidal anti-inflammatory agents are used for C. lndomethacin
C. Inhibiton of mucous production by the gastric mucosa the treatment of following pains, except: D. Butadion
D. Inhibition of regeneratory activity of the gastric A. * Fractures of bones E. Ibuprofen
mucosa B. Headache 20. Gum bleeding arose in the patient after extraction of
E. Inhibition of organotrophic sympathetic influences C. Arthritis the tooth, rom anamnesis it was revealed that the patient
upon the stomach D. Neuritis suffered from rheumatic arthritis, and was treated with the
11. Following signs have developed at the patient after E. Toothache anti-inflammatory agent acetyl-salicylic acid(aspirin).
several days of treatment by the drug with analgesic, 16. A man of 26 years complaints of headache. Previously, Indicate the reason of arisen bleeding.
antipyretic and anti-inflammatory activity: headache, the man suffered from peptic ulcer for the past 4 years. A. *Suppression of synthesis of thromboxane
giddiness, sonitus, nausea, pain in epigastric area. Specify What drug is more preferable in this situation for the relief B. Promotion ol thrombolysis
the drug which could produce similar clinical pattern. of his headache. C. Inhibition ot hemopoesis
A. * Aspirin A.* Paracetamolum D. Decreasing of blood coagulation
B. Naphthyzinum (naphazoline) B. Diclofenac-sodium E. Suppression of synthesis of uric acid
21. A patient with respiratory disease with high
14
temperature took the drug from the group of non-opioid B. Alfa-adrenoreceptors receptors in the CNS
analgesics. This drug exerts predominantly anti- C. Beta-adrenoreceptors D. Excitement of adrenoreceptors and dopamine
inflammatory action which excels the salicilates and D. M-cholinoceptors receptors in the CNS.
butadionum. It is highly absorbed in the intestine and acts E. N-cholinoceptors E. Inhibition of MAO
long. The side effects appear often. Determine this drug. 3. Impairment of movement co-ordination, shivering of 8. A neuroleptic (butyrophenone derivative) was
A. *Indomethacin the hands and sleepiness developed in the patient had been administered to a patient with alcohol psychosis.
B. Meloxicam suffering from schizophrenia under the treatment by Determine this drug.
C. Ortophenum (diclofenac sodium) psychotropic drugs. Indicate this group of drugs. A. Aminazinum (chlorpromazine)
D. Butadion A. *Neuroleptics B. Sulpirid
E. Ibuprofen B. Analgesics C. Diazepam
22. In a 55 years old patient or the 4 ,h day of treatment C. Tranquilizers D. Triftazinum (trifluoperazine)
with indomethacinum gastric hemorrhage developed due D. Antidepressants E. * Haloperidol
to ulcering of gastric mucous membrane. By what is the E. Psyco stimulants 9. Neuroleptoanalgesia was made to a patient because of
ulcerogenic effect of the drug caused ? 4. Introduction of aminazine(chlor-promazine), which was myocardial infarction. What drug from the group of
made to a patient who had been suffering from chronic neuroleptics is used more often in combination with
A. *Decrease of synthesis of prostaglandin
alcholism for elimination of agression and delirium, fentanyl?
E1
caused loss of consiousness in this patient. Indicate the A.* Droperidol
B. Decrease of synthesis of prostaglandin E2
probable reason of this complication. B. Aethaperazinum (perphenazine)
C. Decrease of synthesis of leucotriens
A. *Orthostatic collapse. C. Diazepam
D. Decrease of synthesis of cyclic endoperoxydes
B. Inhibition of the reticular formation. D. Chlozepidum (chlordiazepoxide)
E. Decrease of synthesis of thromboxane
C. Impairment of coronary circulation. E. Sulpind
23. After prolonged taking of a drug in relation with acute D. Suppression of the limbic system. . 10. A patient had been suffering from schizophrenia
respiratory disease a patient began to complain of E. Impairment of oxidative processes in the brain tissue. accompanied by arterial hypertension. A doctor
headache, vertigo, noise in ears, nausea, epigastric pain. 5. Indicate the drug for elimination of the acute psychosis administered neuroleptic possessing expressed hypotensive
Specify the drug that might cause such clinical picture. from the group of derivatives of phenothiazine. activity. Indicate this drug.
A *Acetylsalicylic acid A.*Aminazinum (chlorpromazine) A. *Aminazinum (chlorpromazine)
B. Vitamin С B. Diazepam. B. Risperidone
C. Naphthyzinum C. Haloperidol C. Haloperidol
D. Bromhexinum D. Amitriptylin D. Diazepam
E. Midantanum E. Natrii oxybutyras (oxybate sodium) E. Triftazinum (trifluoperazine)
6. Determine the group of drugs which doesn't cause drug 11. A 35-years-old woman was addressed to a doctor with
NEUROLEPTICS, TRANQUILIZERS. SEDATIVES dependence. complaints of temper tiredness, insomnia, internal tension.
A.* Neuroleptics The doctor had diagnosed neurosis and administered
1. A 40 year-old patient was admitted to the psychiatric B. Tranquilizers tranquilizer (diazepam). Which of the effects of this drug is
clinic in an agitated state. He was aggressive, and C. Barbiturates more important in this situation?
delirated. Indicate the best drug to be given P. Opioid analgesics A. *Anxiolytic
A. *Aminazine (chlorpromazine) E. Psychostimulants -phenylalkylamine derivatives B. Antiemetic
B. Sodium bromide 7. A doctor administered aminazinum (chlorpromazine) to C. Anticonvulsive
C. Diazepam the patient suffering from schizophrenia to eliminate D. Myorelaxation
D. Tinctura Valerianae delirium, hallucinations, to decrease aggression and E. Antipsychotic
E. Peserpinum psychomotor excitement. What is the mechanism of 12. The majority of drugs from this group exert expressed
2. A man with disorders of psychoemotional state and antipsychotic action of aminazinum? sedative - hypnotic and myorelaxation action, that is why
sleeplessness, was treated with diazepamum. Upon which A. Excitation of M-cholinoreceptors in the CNS they decrease concentration and reactions of the cured
receptors this drug influence? B. Stimulation of opioid receptors patients. Determine this group of drugs.
A.* Benzodiazepine receptors C. *Blockade of adrenoreceptors and dopamine A. *Tranquilizers
15
B. Salicylates group of the drugs was administered by the doctor? neuropathologist for neurasthenia for 2 weeks. Patient felt
C. Nootropic agents A. *Bromides better but developed coryza, conjunctivitis, rash, inertia,
D. Sedative B. Valeriana decrease of
E. Ca channel blockers C. Lithium memory. Bromizm was diagnosed. What should be
13. A dentist introduced sibazonum (diazepam) to a 48- D. Tranquilizers prescribed to decrease symptoms?
years old woman before extraction of tooth. Explain the E. Neuroleptics A *Natrium chloride
mechanism of its anxiolytic action. 17. A patient has been taking a mixture prescribed by B Glucose solution 5\%
A. *Agonist of benzodiazepine receptors neuropathologist for neurasthenia for two weeks. The C Asparcam
B. Stimulation of opioid receptors patient feels better but has developed coryza, D Polyglucin
C. Excitation of adrenoreceptors and dopamine receptors conjunctivitis, rash, inertia, decrease of memory. She is E -
in the CNS. diagnosed with bromizm. What drug should be prescribed 22. The patient was treated medically for psychosis for 2
D. Inhibition of α-adrenoreceptors to decrease the symptoms? weeks. Patient's condition improved but rigidity, tremor,
E. Agonist of M-cholinoreceptors A. *Natrium chloride (salty food) hypokinesia developed. Which of the drugs can cause such
14. A patient was addressed to a doctor with complaints of B. Potassium chloride complications?
emotional unstability. feeling of psychoemotional tension C. Polyglucinum A *Aminazine
and fear in meetings with the chief often accompanied by D. Asparcam B Diphenine
significant tachycardia, pain in the heart area, hyperemia E. Glucose solution 5% C Sydnocarb
of the face, headache, hand tremor, sweating. Administer 18. A patient who has been treated in a neural clinic and D Imipramine
the necessary drug. has been taking a sedative for a long time got the following E Chlordiazepoxide
A. *Chlozepidum (chlordiazepoxide) complication: cough, rhinitis, epiphora. What drug caused
B. Aethaperazinum (perphenazine) these disturbances? ANALEPTICS,. PSYCHOSTIMULANTS,
C. Aethtmizolum A *Sodium bromide NOOTROPIC AGENTS
D. Analginum (metamizole) B Diazepam
E. Sulpirid C Valerian 1. During the operation under general anesthesia the
15. What group of drugs is used for the treatment and D Phenazepam patient's respiration was inhibited. Specify, which
prevention of manias? E Reserpine breathing stimulant should be used in this situation
A. * Lithium 19. A patient with myocardial infarction was admitted to without pausing the general anesthesia
B. Sedatives the cardiological department. For pain relief it was decided A. * Aethimizolum
C. Neuroleptics to potentiate fentanyl action with a neuroleptic. Which of B. Coffeinum
D. Tranquilizers the following neuroleptics is the most suitable for C. Bemeqridum
E. Antidepressants neuroleptanalgesia? D. Cytitonum
16. These drugs amplify and concentrate the inhibitory A *Droperidol E. Lobelini hydrochoridum
processes in the brain cortex. They exert sedative action, B Aminazine 3. The patient was admitted to the neurology department
relieve irritability, and do not eliminate the feeling of fear, C Triftazine because of complaints of decrease of memory, mental and
anxiety. Determine this group. D Haloperidol work capacity, sleepiness and vertigo. His symptoms were
A.* Sedatives E Sulpiride connected to a brain concussion, which took place 2 years
B. Tranquilizers 20. A 45-year-old patient suffers from neurosis ago as a result of an automobile accident. What drug
C. Antidepressants characterized by irritability, sleeplessness, motiveless should be indicated to improve his condition.
D. Neuroleptics anxiety. What drug would eliminate all the symptoms? A.* Pyracetamum (Nootropilum)
E. Psychostimulants A *Diazepam B. Natrii oxybutyras (oxybate sodium)
17. A patient was addressed to a doctor with complaints of B Valerian extract C. Sydnocarbum
irritability, insomnia, fatigue. A doctor administered a C Pyracetam D. Coffeinum
sedative drug to him. In two weeks the patient began to D Caffeine sodium benzoate E Cordiaminum (nikethamide)
complain of cough, sleepiness, decrease of memory, E Levodopa 4. Specify indication for use of coffeinum.
phenomena of rhinitis, conjunctivitis, dermatitis. What 21. The patient has taken the mixture prescribed by A. *Sleepiness.
16
B. Arterosclerosis. A. *Nootropic agents (cognitive enhancers) C. Coffeinum
C. Arterial hypertension. B. Analeptics D. Droperidolum
D. Tachycardia. C. Psychostimulants E. Ortophenum (diclofenac-sodium)
E Nausea. D. Antidepressants 3. A doctor administered amitriptyline to a patient with
5.Indicate the drug which posseses analeptic and psycho E. Adaptogens endogenous depression. Explain the mechanism of action
stimulant activity. 13. Specify the characteristic side-effect of analeptics of this drug.
A.*Coffeinum A. *Convultions A. *Inhibition of the neuronal reuptake of noradrenaline
B. Bemegridum B. Bronchospasm and serotonin
C. Aethimizoium C. Hepatitis B. Inhibition of the neuronal reuptake of serotonin
D. Cordiaminum (nikethamide) D. Arterial hypertension C. МАО inhibitor
E. Strychnine E. Gastritis D. Increase of release of noradrenaline and serotonin
6. The patient was addressed to a doctor with complaints 14. Specify the main effect of Piracetam (Nootropil). E. Inhibition of the neuronal reuptake of noradrenaline
of tiredness, decrease of capacity for mental and physical A. *Improves the processes of memorising and 4. A man was addressed to the psychiatrist with complaints
work, worsening of mood. In examination the doctor cognitivity of dreary spirits, feeling of hopelessness and desperation,
revealed arterial hypertension and administered the drug B. Tranqulizing effect tendency to suicide. Determine the group of drugs for the
from the group of psychostimulants. What agent is C. Inhibition upon excitation in the CNS treatment of this patient?
contraindicated in this situation ? D. Stimulation of myocardium activity A.* Antidepressants
A. *Phenaminum (amphetamine) E Deacreases the neuronal stability to hypoxia B. Sedatives
B. Coffeinum 15. An aged patient complains of headache, dizziness, C. Neuroleptics
C. Ginseng quick tiredness, worsening of memory. Anamnesis: D. Tranquilizers
D. Piracetam craniocerebral injury. Medicine of what group should be E. Lithium
E. Sydnocarbum (mesocarbe) prescribed? 5. This group of drug includes psychotropic agents of plant
7. What is the main mechanism of psycostimulant action A *Nootropics origin, These drugs are used in asthenic states after severe
of coffeinum? B Somnific infectious diseases. They increase general vital tonicity of
A.* Blockade of the adenosine receptors C Neuroleptics the organism and its resistance to infectious diseases.
B. Agonist of M-cholinoreceptors D Analgetics These drugs are used in the form of tinctures or liquid
C. Stimulation of opioid receptors E Sedatives extracts. Choose this group of drug.
D. Excitation of adrenoreceptors and dopamine receptors A. *Adaptogens
in the CNS. ANTIDEPRESSANTS. ADAPTOGENS. B. Sedative
E. Inhibition of α-adrenoreceptors C. Psychostimulants
This drug is used for the treatment of disorders of 1. A woman tried to commit suicide, her psychiatrist made D. Antidepressants
memory, cerebral atherosclerosis, after cerebral injuries, the diagnosis of endogenic depression. What drug should E. Nootropic agents
alcohol encepholpathy, dementia. It improves the be given for her treatment? 6. This drug has stimulating action promoting synthesis of
processes of memorizing and cognitively. Determine this A. *Amitriptyllinum proteins and ATP, increases the capacity for physical and
drug. B. Nootropilum psychical work,. It is used for a long time and it shouldn't
A. *Piracetam C. Sydnocarbum (mesocarbe) be taken before sleep.
B. Amitriptylinum D. Aethimizoium A.*Tincture of Ginseng
C. Paracetamol E. Coffeinum B. Tincture of Valeriana
D. Bemegridum 2. The patient addressed to a doctor with complaints of bad C. Barboval
E. Sibazonum (diazepam) mood, that was accompanied by expressed sedative action, D. Nialamide
12. A patient with traumatic encephalopathy was admitted feeling of fear, anxiety. The clinical examination revealed E. Natrium bromide
to the psychical depression. What drug should be administered to
neurological department with complaints of disorders of this patient? THE AGENTS INFLUENCING THE BLOOD
memory, intellect, headache, vertigo. Choose the group of A. *Amitriptylinum COAGULATION, ERYTHRO- AND
drugs for the treatment of the patient. B. Ammazinum (chlorpromazine) LEUCOPOIESIS.
17
1. The patient was admitted to the traumatologic 6. Specify the thrombolytic agent which belongs to with atherosclerosis. Complex of her pharmacotherapy includes
department due to fracture of chin bones, damages of soft direct action plasma proteins. drug of vitamin E & С. Which pharmacological effect of
tissues and massive bleeding. Examination revealed A. *Fibrinolysin these drugs is the main for treatment of this disease?
paleness of the skin, pain in palpation of area of trauma, В. Streptokinase A. * Inhibition of free-radical oxidation of lipids
swelling of the skin, bleeding on the whole surface of the C. Contrycalum B. Increase of gonadotropic hormones synthesis in the
wound. Specify a drug for local use to stop the bleeding. D. Urokinase pituitary body
A. * Thrombinum E. Streptodecase С. Decrease of glucocorticoids release by adrenals
B. Calcii chloridum 7. Inhibition of leukopoiesis is observed in a 43 years old D. Strengthening of the antitoxic function of the
С. Vikasolum roentgenologist. The amount of leukocytes 3,5*109/l. liver
D. Aminocapronic acid Specify the agent to be used for correction of E. Improvement of the coronary circulation
E. Ambenum leukopoiesis.
3. A patient who had been taking л vitamin drug for the
2. Drugs delaying blood coagulation (anticoagulants) are A. *Pentoxylum
prevention of cerebrovascular spastic reactions began to
used for prevention and treatment of thrombosis. Specify B. Ferroplexum
complain of unpleasnt feelings: hyperemia of the face and
an anticoagulant which antagonist is protamine sulfate. С. Hemostimulinum
upper part of the body, vertigo, feeling of blood influx into
A. * Heparinum D. Cvanocobalaminum
the head. What drug may cause these side-effects
B. Meodicoclmarin E. Ascorbic acid
A. *Nicotinic acid
C. Syncumarum 8. Before tooth extraction a patient was advised to take a
В. Tocopheroli acetas
D. Phenilinum certain drug for haemorrhage prevention. What drug
C. Nicotinamidum
E. Sodium hydrocitrate was advised?
D. Thiamin/ bromidum
A *Vicasolum
3. A patient was delivered to a hospital with complaints of E. Calcii pangamas
B Heparin
loss of appetite, decrease of body weight, fatigue, pain
C Asparcam 4. A patient had been taking vitamin D for a long time for
around the epigastric area. Examination of the blood
D Magnesium sulfate treatment of rickets. Soon the signs of vitamin D
revealed megaloblastic anemia. Specify the main agent for
E Dimedrol intoxication developed: loss of appetite, nausea, headache,
the treatment of this disease.
9. A patient ill with essential hypertension was fatigue, increase of body temperature, etc. What vitamin
A. *Cyanocobalaminum
recommended a drug that prevents thrombosis. It is to be decreasing the toxic influence of vitamin D should be
B. Ferri lastas
taken parenterally. What drug is it? administered?
C. Folic acid
A *Heparin
D. Fercovenum A *Vitamin A
B Amben
E. Coamidum B. Vitamin С
C Protamine sulfate
C. Vitamin В12
4. Specify the antagonist of the anticoagulants with D Neodicumarin
D. Vitamin PP
indirect action. E Syncumar
E. Vitamin B2
A. *Vikasolum
B. Fercovenum VITAMINS 5. A 55 year old patient was admitted to the
С Pentoxylum haemotological department with signs of acute anemia.
D. Protamini sulfas 1. A 64 year old woman suffers from night blindness After laboratory examination megaloblastic hyperchromic
E. Contrykalum (hemeralopia). What vitamin drug should be anemia was diagnosed, which drug must be administered
5. Specify the coagulant agent available for local use only recommended? to the patient first of all?
(to stop bleedings from small blood vessels). A. * Retinolum A. *Cyanocobalaminum
A. *Hemostatic sponge B. Riboflavin B. Hemostimulinum
B. Vikasolum С. Tocopherolum C. Ferroplexum
С Calcii chtoridum D. Pyridoxinum D. Ferrum-Lek
D. Fibrinogen E. Ascorbic acid E. Folic acid
E. Aminocapronic acid 2. A 58 year old woman suffers from cerebral 6. A 55 year old patient suffering from hyperchromic
18
anemia obtained long-term treatment with vitamin В12 11. A patient suffers from chronic alcoholism with the D. Cyanocobaiamine
parenterally. Why the parenteral way of introduction of following symptoms: pain in arms and legs, impairment of E. Folic acid
vitamin B12 is prefered more than the peroral way? skin sensitivity, muscle weakness, edemas and increased
A.*It isn't absorbed in the intestine in oral introduction amount of pyruvate. Which vitamin drug should be 16. The woman of 25 years who anted to get thin was on a
due to deficit of gastromucoproiein prescribed to the patient? diet hich consisted of 2 cups of coffee ithout sugar, 3
B. It is faster absored A. * Thiamine crackers from white bread, 2 sausages or 2 eggs in a day
С It is longer circulated in the blood B. Ergocalciferol within 1,5 months. The mass of a body has dropped on 5
D. It is faster penetrated to the bone marrow C. Retinol kg, but there were developed strong headaches, often
E. it isn't destroied in the liver D. Rutin nasal bleedings, bleeding, sickness of gums, shaking of
7. A patient has diarrhoea, dementia and dermatitis. What E. Vikasoium (Menadione) teeth, -.helling of sfcin and shedding of hair.What vitamin
vitamin has to be included into the complex drug is expedient in this (ase?
12. 55 years old patient was admitted to the
pharmacotherapy? A. *Ascorbic acid
hematological department with acute anemia: RBCs - 1,5
A. *Nicotinic acid B. Folic acid
x 1012/litre; Hb - 80g%, colour index - 1,3. Hyperchromic
B. Thiaminum C. Retinol acetate
anemia was diagnosed. Which drug should be
C. Cyanocobalaminum D. Cyancobalamine
administered for treatment of this disease?
D. Panthotenic acid E. Vikasolum (Menadione)
A. *Cyanocobalamine
E. Riboflavinum 17. The patient who was treated by a vitaminic drug for
B. Ferroplexum
8.There is an inhibited coagulation in the patient with bile С. Hemostimuline prophylaxis of vasospasms of the brain, has developed
ducts obstruction, bleeding due to the low level of D. Ferrum-lek complaints of the unpleasant sensations related to taking
absorbtion of a vitamin. What vitamin is in deficiency? E. Folic acid of this medicine: reddening of the face and the upper half
A. *Vitamin К of a trunk, giddiness, sence of flush of blood to a head. For
13.Which of the acids below decreases permeability of what drug the specified side-effects are characteristic?
B. Vitamin D
connective tissue structures, possesses antioxidant
С. Carotene A. *'Nicotinic acid
activity due to ability to be transformed from the
D. Vitamin A B. Nicotinamidum
oxydized form into reduced and on the contrary?
E. Vitamin E. C. Thiamine bromide
A. * Ascorbic acid
9. A patient with diabetes mellitus is treated by injections D. Tocoferol acetate
B. Hydrochloric acid
of vitamin B1 to eliminate metabolic acidosis. Which E. Calcium pangamate
C. Mefenamic acid
biochemical mechanism of action of vitamin B1 ensures
D. Aspirin (acetylsalicylic acid) 18. Bioflavonoids (rutin, quercetin) possess all listed
the positive effect?
E. Aminocaproic acid below pharmacodynamic effects, except for:
A. * Activation of dehydrogenases of the Krebs cycle
B. Activation of adenylate cyclase 14. For synthesis of the basic substances of connective A. * Dilation of capillaries
С. Blockade or phosphodiesterase tissue (mucopolysaccharides and collagen) an essential B. Antioxidant activity
D. Acceleration of acetylcholine synthesis agent is: C. Inhibition of hyaluronidase activity
E. Acceleration of adrenaline synthesis A. * Ascorbic acid D. Decrease of permeability of capillary wall
B. Nicotinic acid E. Protections of ascorbic acid, assistance to its
10. Metabolic acidosis arose in a patient due to
С. Folic acid transport and accumulation
impairment of carbohydrate metabolism and
D. Salicylic acid
accumulation of ketoacids in the organism, indicate the 19. 64 years old woman suffers from hemeralopia
E. Acetylsalicylic acid
vitamin drug which promotes its elimination due to (disturbance of vision in darkness). What vitaminic drug
decrease concentration of ketoacids. 15. Radiation therapy is performed to the patient. What should be recommended her first of all?
A. *Thiamine vitamin drug with antioxidant properties is necessary for A. *Retinoli acetate
B. Pyridoxins administration to increase stability of tissues in this case? B. Tocoferol acetate
С. Folic acid A. * Ascorutinum C. Pyridoxin
D. Riboflavine B. Vitamin B6 D. Ascorbic acid
E. Ascorbic acid С Thiamine chloride
19
E. Riboflavin 25. The dentist administered to his patient liposoluble C Pyridoxine hydrochloride
vitamin with antioxidant activity for treatment of D Ascorbic acid
20. What vitamin promotes growth and development of
parodontitis. Specify this vitaminic drug. E Nicotinic acid
epithelial cells, including epidermal ones?
A. * Tocoferol acetate 31. A patient suffers from vision impairment - hemeralopy
A. *Retinol (night blindness). What vitamin preparation should be
B. Ascorbic acid
B. Ergocalciferol administered the patient in order to restore his vision?
C. Rutin
C. Ascorbic acid A *Retinol acetate
D. Nicotinic acid
D. Nicotinic acid B Vicasol
E. Ergocalciferol
E. Lipoic acid C Pyridoxine
26.A doctor administered tocoferol acetate to a patient with D Thiamine chloride
21. 39 years old patient suffers from hyperkeratosis,
ischemic heart disease. What effect of a drug does the E Tocopherol acetate
disturbance of vision in darkness, frequent infectious
doctor expect?
diseases. What vitaminic drug should be administered for
A. *Antioxidant PLASMA SUBSTITUTES. AGENTS FOR
treatment .
B. Spasmolytic CORRECTION OF WATER-SALT BALANCE
A. *Retinol acetate 1. A patient was admitted to the intensive care unit after
С. Hypotensive
B. Pyridoxin taking a large dose of Phenobarbitalum with the purpose of
D. Increase of oxygen delivery to myocardium
С. Riboflavin suicide. Investigation of the patient revealed respiratory
E. Positive inotropic
D. Ergocalciferol olic acidosis. What drug should be used for the correction
E. Tocoferol acetate 27. What enzymatic drug is used with the purpose of
of acid-base state?
22. What vitamin is formed in skin under dropping of density and rising of permeability of
A * Trisaminum
influence of ultraviolet radiation? connective tissue structures?
B. Sodium chloride
A. *Cholecalciferol A. *Lidase C. Calcium chloride
B. Ascorbinic acid B. Lipase D. Ammonium chloride
C. Retinole acetate С. Cocarboxylase E. Potassium chloride
D. Calcium pantotenate D. Cholines terase
E. Riboflavin E. Amylase 2. During treatment of the patient with digitoxin,
28. In order to accelerate healing of a radiation ulcer a extrasystoles, muscle weakness, diarrhea, vomiting, and
23. To the child with signs of rachitis the pediatrist and the impairment of vision developed. What drugs should be
vitamin drug was administered. What drug is it?
"dentist administered a liposoluble vitamin which used to eliminate these signs of intoxication?
A *Retinol acetate
influences an exchange of phosphorus and calcium in an
B Retabolil A. * Drugs of potassium
organism, promotes sedimentation of calcium in bone
C Prednisolone B. Drugs of calcium
tissue and dentins. Determine a drug.
D Levamisole С. Iron preparations
A. *Ergocalciferol E Methyluracil D. Drugs of sodium
B. Tocoferol acetate 29. A 64 year old woman has impairment of twilight vision E. Drugs of magnesium
С. Retinoli acetate (hemeralopy). What vitamin should be recommended in
D. Vikasolum (Menadione) 3. What agent acts as magnesium ions' antagonist and is
the first place?
E. Thyreoidinum used in overdosage of parenteral introduction of
A *Vitamin A
magnesium sulphatis?
24. Parasthesia, xeroderma arid sticking out of fontanel B Vitamin B2
A. *Calcium
are observsed at the 6 months child under the treatment by C Vitamin E
B. Potassium
a vitaminic drug. Specify this drug. D Vitamin C
C. Sodium
A. *Ergocalciferol E Vitamin B6
D. Iron
B. Pyridoxine 30. A patient presents with twilight vision impairment.
E. Bromine
С. Riboflavin Which of the following vitamins should be administered?
D. Retinoli acetate A *Retinol acetate 4. Specify the drug which eliminate both intra- and
E. Tocoferol acetate B Cyanocobalamin extracellular acidosis.
20
A. *Trisaminum day. Recently he has started to complain of painful dyspeptic disorders, appearance of a tumescence on the
B. Natrii hydrocarbonas convulsions of skeletal muscles of lower extremities. front surface of a neck. Analysis of the blood revealed
С. Natrii lactas What agent may be used or improvement of patient's leukopenia and agranulocytosis. With what antithyroid
D. Ammonii chioridum state? drug was the patient treated?
E. Natrii hydroxydum A * Panangin A. * Mercazolilum
5. Specify the drug of first choice to be administered in a В. Ergocatciferol B. Diiodotyrosine
7-year old child with multiple caries. С. Calcitonin С. Iodine
A. *Calcii glycerophosphas D Diazepam D. Radioactive iodine
B. Calcii gluconas E. Aminazine (chlorpromazine) E. Potassium perchlorate
С. Calcii chloridurn 10. The patient was admitted to the hospital with signs of 2. The patient suffering from diabetes mellitus
D. Calcii hydroxydum dehydration. The doctor has immediately administered him complained of flaccidity, apathy, difficulty of respiration.
E. Calmecinum intravenous infusion of Sodium chloride. At what situation Investigation of the patient revealed: confused
6. The worker who several days ago started working in is necessary to use this drug? consciousness, paleness and dryness of the skin and
foundry, was addressed to the doctor with complaints of A. *Cholera mucous membranes, sunken eyes, sharp pointed features
headache, nausea. Due to excessive diaphoresis he drank B. Toxicosis of pregnancy of the face. The blood pressure is 100/60 mmHg. Pulse is
nearly 5 liters of tap water per day. What drug will C. Osteoporosis weak and frequent. Odour of acetone from the mouth.
promptly and effectively eliminate the specified signs and D. Edemas The diagnosis of hyperglycemic coma was given. Which
normalize state of the worker? E. Arthritis drug should be introduced for the elimination of the given
state?
A. * Sodium chloride 11. Convulsions and laryngospasm have developed at the
B. Decamevitum A * Insulin
patient with hypoparathyrosis. Laboratory examination
С Aspirin (acetylsalicylic acid) B. Glibutidum
revealed significant decrease of calcium ions
D. Analginum (methamizole) С Bucarbanum
concentration in the blood and slight elevation of pH.
E. Pentalginum D. Protamine-zinc-insulin
Which drug should be introduced for correction of
E. Butamidum
7. Ketoacidosis and dyspnoe are observed at the patient metabolic alkalosis?
with non-compensated diabetes mellitus. Which drug A. *Ammonium chloride 3. Hypoglycemic state arose in the patient who had been
should be used for normalization of patient’s state? B. Trisamine suffering from diabetes mellitus and was treated with
A. *Sodium hydrocarbonate C. Magnesium oxide insulin's drug of prolonged action. Indicate the endocrine
B. Naloxone D. Aluminium hydroxide drug which should be introduce to eliminate this plate.
C. Bemegride E. Sodium hydrocarbonate A. *Glucagonum
D. Pananginum В Hydrocortisone
12. Which of plasma substitutes listed below circulates in
E. Ammonium chloride C. Triamcinolonum
the blood for a long time?
D. Noradrenaline
8. It is necessary to eliminate ketoacidosis at the patient A. * Rheopolyglucinum
E. Prednisotonum
with sugar diabetes complicated by hyperglycemic coma. B. 5% glucose solution
What solution, being the intracellular buffer, can be С. 0.9% solution of Sodium chloride 4. Specify the hormonal agent which is used in diabetes
administered? D. Ringer-Locke solution insipidus.
A. * Trisamine (trometamol) E. Polydesum A. *Adiurecrinum
B. Solution of Sodium hydrocarbonate B. Oxytocin
С. Solution of Sodium lactate C. Insulin
D. Neohemodesum HORMONAL DRUGS (1)
D. L-thyroxin
E. Ringer’s solution E. Prednisolonum
1. After prolonged treatment of thyrotoxicosis with an
9.Ths patient suffering from ollagenosis has been treated antithyroid drug, suppressing synthesis of thyroid 5. In a woman during labor, weakness of labor activity is
for a long time by Prednisolone in a dose of 30 mq per hormones in a thyroid gland, the patient complained of the determined. What hormone drug should be introduced for
21
stimulation of myometrium contractions? 10. A 19-year-old female suffers from tachycardia in thyreoidectomy. By usage of what drug is it possible to
A. *Hormone of n. supraopticus of the resting condition, weight loss, excessive sweating, eliminate this state?
hypothalamus (oxytocin) exophthalmos and irritability. What hormone would you A. *Calcium chloride
В. Follicle stimulating hormone expect to find elevated in her serum? B. Tubocurarine
С. Prolactin A. *Thyroxine С. Calcitrinum
D. Luteinizing hormone B. Cortisol D. Triiodothyronine
E. Hormone of n. paraventricularis of the С. ACTH E. Ergocalciferol
hypothalamus (vasopressin) D. Mineralocorticoids 16. At investigation of a sick woman hyperactivity of
E. Insulin thyroid gland is revealed. What drug should be
6. After removal of the thyroid in a patient the attacks of
convulsions are observed. What drug needs to be 11. Specify the hormones entering into composition of administered to her?
administered? pituitrin. A. * Mercazolilum (methimazole)
A. *Parathyreoidin A. *Oxitocin and Vasopressin B. L-thyroxine
B. Somatotropin B. Oxytocin and progesterone С. Triiodothyronine
С. Insulin C. Oxytocin and oestradiol D. Lugol’s solution
D. Prednisolonum D. Vasopressin and oestradiol E. Thyreoidinum
E. L-thyroxin E. Vasopressin and progesterone 17. A drug of iodine is administer, to the patient,
7. A patient was treated for a long time by a 12. To the patient after a subtotal resection of thyroid suffering from hyperthyroidism of an initial stage. What
glucocorticoid drug. After sharp stopping of taking a drug gland the drug of replaceable therapy is administered. effect underlies antithyroid activity of this drug?
following complaints were arisen: undue fatigability, Specify this agent. A. * Oppression of release by pituitary body of
emotional lability, sleeplessness, headache, decrease of A. * L thyroxine sodium thyrotropic hormone
appetite, nausea. The syndrome was diagnosed. What B. Potassium perchlorate В. Oppression of activity of the enzymes participating
drug should be administered for correction of this state? C. Mercazolilum in synthesis of iodine-containing hormones of
A. *ACTH D. Rifathyreline thyroid gland
B. Glucocorticoids E. Potassium iodide С. Activation of production by hypothalamus of
С. Adrenaline thyroliberine
13. Impairment of enamel and dentine formation is D. Inhibition of the process of iodation of hormones’
D. Corticosteroids diagnosed in a child owing to the under content of
E. Mineralocorticoids molecules
calcium ions in blood. What hormonal drug can be E. Destruction of tissue of thyroid glan
8. Specify the drug of posterior pituitary hormone applied administered to correct this state? 18. Radioiodine (J 131) is administered to a 65 years old
to stimulation of labor activity of uterus. A. *Calcitonin patient suffering from Basedow’s disease, considering
A. *Oxytocin B. Thyroxine inefficiency of treatment by the basic drugs and
B. Dinoproston C. Somatotropinum impossibility of a surgical intervention. What is the
С. Pachycarpin D. Prednisolone mechanism of action of this agent?
D. Dinoprost E. Cortisone A. * Causes destruction of follicles of thyroid gland
E. Salbutamol 14. Mercazolilum (methimazole) had been administered to B. Inhibits functions of thyroid gland
9. A 25 years old woman was delivered to a maternity the patient, suffering from thyrotoxicosis. What effect С Oppresses synthesis of hormones of thyroid gland
home for delivery. Due to uterine inertia a doctor underlies antithyroid activity of a drug? D. Accelerates excretion of iodides from thyroid gland
administered her hormonsil agent. Indicate this drug. A. *Decrease of thyroid hormones production E. Blocks entering of iodine in thyroid gland
A. *Oxytocin B. Depression of production of thyrotropic hormone 19. Numbness of extremities and paresthesia were
B. Retabolilum С. Depression of uptake of iodine by thyroid gland developed at the 52 years old patient soon after the
С. Progesterone D. Destruction of cells of thyroid gland operation of thyreoidectomy, hypocalcemia is laboratory
D. Gonadotropin chorionic E. Oppression of metabolism in thyroid gland diagnosed. What hormonal drug is necessary for
E. Testosterone 15. Convulsions were developed at the patient after administering?
22
A. * Parathyroidin patient? D. Captopril
B. Thyreoidinum A. *Glucose E. Nifedipine
С. Calcitrinum B. Insulin 28. A 60 years old patient has been suffering within 9
D. Thyroxine С. Lydasum years from diabetes mellitus and is treated by insulin. 10
E. Triiodothyronine D. Somatotropinum days ago his doctor administered to him hypotensive agent
20. The patient after removal of thyroid gland suffers from E. Noradrenaline to treat arterial hypertension. In an hour after last
attacks of convulsions. What drug is necessary for 25. A 60 years old patient has been suffering within 9 introduction of this drug hypoglycemic coma has
administering in this case? years from diabetes mellitus and is treated by insulin. 10 developed. Which of the numbered drugs should be used
A. *Parathyroidin days ago his doctor has administered to him beta for treatment of this state?
B. Insulin adrenoblocker propranolol (anaprilinum) due to arterial A. * Glucose
С. Prednisolone hypertension. In an hour after last introduction of В. Insulin
D. Thyroxine propranolol hypoglycemic coma has developed. Indicate a C. Noradrenaline
E. Somatotropin reason of arising of hypoglycemia in this case? D. Natrii hydrocarbonas
21. Insulin was introduced to the patient, suffering from A. * Oppression of glycogenolysis E. Bemegride
diabetes melliyus. What is the main mechanism of action B. Prolongation of action of insulin
of this agent? 29. The state of the patient, suffering from diabetes
C. Decrease of release of glucagon mellitus was worsened after the injection of insulin with
A. * Activation of glucose transport from blood to D. Magnification of bioavailability of insulin long action duration: general asthenia, cold sweat, tremor
tissues E. Diminution of absorption of glucose of extremities developed hypoglycemic coma was
B. Oppression of glycogenesis
26. A 60 years old patient has been suffering within 9 diagnosed. Intravenous injection of glucose solution didn’t
C. Inhibition of transport of amino-acids
years from diabetes mellitus and is treated by insulin. 10 cause improvement of the state. What drug should be used
D. Activation of synthesis of triglycerides
days ago his doctor has administered to him beta- in this situation?
E. Activation of glycogen synthesis
adrenoblocker propranolol (anaprilinum) in connection A. * Adrenaline
22. Hyperglycemic coma is developed at the patient, with arterial hypertension. In an hour after last B. Insulin
suffering from diabetes mellitus. What drug should be introduction of propranolol he has felt asthenia, darkening C. Isadrinum (isoprenaline)
administered in this situation? in eyes, arterial pressure was depressed and in some D Dobutamine
A. *Insulin of short action minutes the patient has lost consciousness. Indicate the E. Dopamine
B. Insulin of long duration of action condition which has developed at the patient.
30. The 56-years patient complained of thirst and frequent
С. Drug from the group of biguanide derivatives A. * Hypoglycemic coma emiction. After investigation in endocrinology the
D. Insulin of intermediate duration of action B. Hyperglycemic coma diagnosis of diabetes mellitus was established, and
E. Drug from the group of sulfonylurea derivatives C. Cardiogenic shock Butamidum (carbutamide) was administered to him.
23. Which agent from listed below should be introduced D. Sharp impairment of cerebral circulation Determine the mechanism of action of this agent.
for treatment of hyperglycemic coma? E. Allergic reaction
A. * Stimulates beta-cells of pancreatic islets of
A. *Insulin 27. A 60 years old patient has been suffering within 9 Langerhans
B. Metformin years from diabetes mellitus and is treated by insulin. 10 B. Promotes utilization of glucose by tissues of an
С. Protamin-Zincum-insulin days ago his doctor has administered to him hypotensive organism
D. Chlorpropamide agent to treat arterial hypertension. In an hour after last С. Facilitates glucose transport through cellular
E Bytamidum (Carbutamide) introduction of this drug hypoglycemic coma has membranes
developed. Which of the numbered drugs could cause this D. Oppresses beta-cells of pancreatic islets of
24. To the patient, suffering from diabetes mellitus the
complication? Langerhans
nurse has wrongly introduced almost double dose of
A.* Propranolol (anaprilinum) E. Increases a level of glucose in a blood
insulin. In 2 hours sweating, tachycardia, convulsions and
loss of consciousness has developed. Hypoglycemic coma B. Prazozin 31. What drug stimulating release of endogenic: insulin is
is diagnosed. What drug should be introduced to the С. Verapamil a derivative of sulfonylurea with average action duration
23
(8-24 hours)? 36. The patient of 45 years complains of constant thirst hyperglycemia and glucosuria. A doctor administered him
A. * Butamidum (carbutamide) and expressed polyuria. The level of glucose in blood a medication for internal use. What medication is it?
B. Buforminum plasma is normal, and in urine glucose misses. What drug A Glibenclamid
C. Metforminum should be chosen for his treatment? B Furosemide
D. Gilipizid A. *Adiurecrinum C Oxytocin
E. Chlorpropamide B. Desoxycorticosterone D Pancreatine
С Insulin E Corglycon
32. Specify a drug, which is suitable to be used in patients D. Hydrocortisone 42. An elderly female patient suffers from the type 2
suffering from diabetes mellitus with lowered ability of E. Prednisolone diabetes mellitus accompanied by obesity, atherosclerosis,
pancreas to produce insulin coronary artery disease. Basal hyperinsulinemia is also
A. * Butamidum (carbutamide) 37.What drug oppresses absorption of glucose in small
present. What treatment would be the most appropriate?
B. Insulin intestine?
A *Glibenclamid
С Adrenaline A. * Acarbose
B Insulin
D. Glucagon B. Insulin
C Retabolil
E. Calcitrinum С. Butamidum (carbutamide)
D Lovastatin
D. Metformin
33. The patient of advanced age addressed to the doctor E. Glucose E Amlodipine
with complaints of dryness in a mouth, polyuria, weight
loss. At examination hyperglycemia and glycosuria are 38. Acute bronchitis is arisen in a patient suffering from
revealed. It is diagnosed: diabetes mellitus, II type (non diabetes mellitus. What antimicrobial drug is undesirable HORMONAL DRUGS (I1)
insulin dependent), an intermediate degree of gravity. for administering to this patient? 1. A patient ill with collagenesis has been taking
Which of the numbered drugs should be administered to A. *Biseptol (co-trimoxazole -combined sulfonamide) prednisolone for a long time. Hypokaliemia development
the patient? B Amptcillin (antibiotic) caused spastic pain of skeletal muscles. What medication
С. Tetracycline (antibiotic) should be used in order to correct potassium exchange?
A. *Butamidum (carbufamide)
D. Azithromycin (antibiotic) A *Panangin
В. Insulin
E. Cefotaxim (antibiotic) B Dithylinum
С. Acarbose
39. A 56-year-old patient with complains of thirst and C Diazepam
D. Glucose
frequent urination was diagnosed to have diabete mellitus D Noshpa
E. Adrenaline
and butamin was prescribed. What is the mechanism of E Thyrocalcitonin
34. At examination of 70 years old patient hyperglycemia action of this medicine? 2. Patient was on glucocorticoids for a long time,
is revealed. What drug is expedient for treatment of this A *It stimulates beta-cells of Langergans' islets discontinuation of usage caused exacerbation of the illness,
state which is used oraly? BIt helps to absorb the glucose by the cells of the organism decreased BP, weakness. How can you explain it?
A. *Glibenclamide tissues A *Insufficiency of adrenal glands
B. Mercazolllum CIt relieves transport of glucose through the cells' B Adaptation to the medicine
C. Parathyroidin membranes C Sensitization
D. Iinsulin D It inhibits alpha cells of Langergans' islets D Hyperproduction of ACTH
E. Cortisone E It inhibits absorption of glucose in the intestines E Cumulation
40. Examination of a 70 year old patient rrevealed insulin- 3. Testosterone and it's analogs increase the mass of
35. Indicate the synthetic antidiabetic drug from the group dependent diabetes. What drug should be administered? skeletal muscles that allows to use them for treatment of
of biguanide derivatives. A *Glibenclamid dystrophy. Due to interaction of the hormon with what cell
A. *Metformin B Insulin substance is this action caused?
B. Acarbose C Mercazolilum A *Nuclear receptors
С Butamidum (carbutamide) D Parathyroidin B Membrane receptors
D. Glibenclamide E Cortisone C Ribosomes
E. Glipizid 41. Examination of a 60 y.o. patient revealed D Chromatin
24
E Proteins- activators of transcription of the face and neck), disorder of menstrual cycle. What D. Cortisone
4. A patient ill with neurodermatitis has been taking drug the patient took? E. Testosterone
prednisolone for a long time. Examination revealed high A. * Prednisolone 13. A patient, 42 years old, took glucocorticoids in relation
rate of sugar in his blood. This complication is caused by B. Indornethacin with rheumatoid arthritis. In 3 weeks the signs of arthritis
the drug influence upon the following link of carbohydrate C. Butadionum (Phenylbutazone) were almost eliminated and the patient stopped taking Им
metabolism: D. Synaflanum (Fluoclnolone) drugs. But a day after there was a relapse, and the new
A *Gluconeogenesis activation E. Beclomethasone attack was significantly more severe than at the beginning
B Glycogenogenesis activation
9. The 60 years old woman who had been suffered from of the disease. What's the reason of the developed
C Intensification of glucose absorption in the bowels
arthritis of knee joint was treated for a long time by complication?
D Inhibition of glycogen synthesis
Dexamethasone. Indicate the mechanism of anti- A. *Decrease of release of glucocorticoids
E Activation of insulin decomposition
inflammatory action of this drug? B. Increase of methabolism of glucocorticoids
5. Continious taking of a drug can result in osteoporosis,
A. *Blockade of phosphlipase A2 С. Slowing down of transport of glucocorticoids
erosion of stomach mucous membrane, hypokaliemia,
В Blockade of cyclooxygenase-1 D. Speeding-up of elimination of glucocorticoids
retention of sodium and water, reduced content of
С. Blockade of cyclooxygenase- 2 E. Increase of glucocorticoids' receptors adaptation
corticotropin in blood. Name this drug:
A *Prednisolone D. Blockade of folate synthase 14. A woman, 28 years old, was admitted to a hospital in
B Hydrochlorothiazide E. Blockade of folate reductase relation with danger of miscarriage. Earlier she had two
C Digoxin cases of preliminary labor. Specify the drug of the
10. The patient suffering from arthritis has been treating hormone of corpus luteum that needs to be introduced in
D Indometacin
for several months by glucocorticoid agent this case.
E Reserpine
Dexamethasone. Recently he has begun to complain of
6. Treatment by anti-inflammatory drugs was administered nausea, frequent stomach aches. Clinical examination A. *Progesterone
to the patient with rheumatic endocarditis. After a while has revealed ulcer of the stomach. What is the B. Praegninum
hyperglycemia was him. What group of drugs is capable to mechanism of ulcerogenic action of glucocorticoids? С. Diazepam
provoke such side-effect? A *Impairment of microcirculation in gastric mucousa D. Magnii sulfas
B. Blockade of prostaglandin synthase E. Tocopherol
A. * Glucocorticoids
B. Antibiotics of the group of Penicillins C. Inhibition of protein synthesis due to activation of 15. A patient had been taking glucocorticoids for a long
C. Ascorbic acid gluconeqgenesis time. When the drug was withdrawn he developed the
D. Sedatives D. Initiation of n. vagus symptoms of disease aggravation, decreased blood
E. Nonsteroid anti-inflammatory agents E. Blockade of gastrin synthesis pressure arid weakness. What is the reason of this
11. A patient suffers from systemic inflammatory condition?
7. Many kinds of pathological states (inflammation,
affection of connective tissue. Indicate the drug A. *Appearance of adrenal insufficiency
pulmonary edema, shock of different origin) are
inhibiting all phases of inflammation which should be B. Sensibilization
accompanied by violation of permeability of vessels.
administered to the patient? С. Habituation (tolerance)
Which of the listed below drugs can be used for
A. *Dexamethasone D. Hyperproduction of ACTH
elimination of this reaction at any of the termed states?
В. Butadionum (Phenylbutazone) E. Cumulation
A. * Prednisolone
B. Indomethacinum С. Contrycal (Aprothtin) 16. A patient with severe inflammatory disease had been
C. Dimedrolum (diphenhydramine) D. Indomethacine treated by prednisolone during 10 months. Due to
D. Aspirin (acetylsalicylic acid) E. Diclofenac-sodium improvement of patients state the doctor has reduced a
E. Beclomethasone dose of prednisolone and added corticotropin. What
12. Specify a synthetic analogue of glucocorticoid
hormones. purpose has he pursued, administering corticotropin?
8. The woman of 33 years on a phone of long-lasting A. *Stimulation of suprarenal glands activity
medicamental therapy of chronic polyarthritis, began to A *Prednisolonum
B. Potentiotion of drugs effects
mark herself increase of arterial pressure, change of В. Adrenaline
C. Replacement of prednisolone
allocation of fatty tissue (accumulation mainly in the area C. Pituitrinum
25
D. Decrease of side-effects of prednisolone В Diclofenac-sodium D. Hydrocortisone
E. Prophylaxis of tolerance to prednisolone C. Ibufrofen E. Prednisolone
D. Indomethacin
17. The patient's state requires introduction of 27. Glucocorticoid agent (Prednisolone) had been
E. Butadionum (phenylbutazone)
glucocorticoids. How is t necessary to administer them administered to the patient of 42 years who suffered from
taking into account physiological change of these 22. The man of 50 years suffers from tuberculosis of the rheumatoid arthritis, in 3 weeks patient’s state had
hormones' content in a blood? skin. Which of numbered drugs, used for treatment of improved and he had discontinued taking of the drug.
A. *2/3 of the dose in the morning, the rest in the dermal diseases, is contra-indicated to this patient? However in a day his condition was worsened. What was
afternoon A. * Prednisolone the reason of this complication?
B. All dose in the evening B. Tetracycline A. * Production of glucocorticoids had dropped
С. All dose in the morning С. Sulfur ointment B. Transport of glucocorticoids was inhibited
D. Ergocalciferol С. Elimination of glucocorticoids had been accelerated
D. Evenly within day
E. Retinol acetate D. Adaptation of receptors to glucocorticoids had
E. 2/3 of the dose in the evening, the rest in the morning
23. What drug cannot be used at infectious affections of strengthened
18. Indicate the mechanism of antiinflammatory action of skin? E. Metabolism of glucocorticoids had strengthened
Prednisolone.
A. * Prednisolone ointment 28. After long-term treatment by glucocorticoid agent this
A. * Inhibition of phospholipase A2 activity B. Gentamvcin ointment
B. Inhibition of transcription drug was abolished, but patient’s state was worsened:
С Tetracyclin ointment exacerbation of the current disease, decrease of arterial
C. Inhibition of COX activity D. Yellow mercury ointment
D. Inhibition of translation pressure; asthenia had developed. Indicate a reason of the
E. Brilliant green arisen condition.
E. Stimulation of COX activity
24. The daily dose of what drug should be parted on A. * Insufficiency of suprarenal glands
19. A doctor had administered several drugs (anti- unequal portions according to phases of a circadian B. Drug tolerance
inflammatory, antibacterial and so on) to a patient who rhythm? С. Sensibilization
suffered from rheumatic carditis. After a while A. *Dexamethasone D. Hyperproduction of ACTH
hyperglycemia has arisen at him. What group of drugs is B. Indomethacin E. Cumulative action
(capable to cause such side-effect? С. Ibufrofen 29. The patient of 60 years has tolerated mastectomy.
A. * Glucocorticoids D. Diclofenac-sodium After a course of radiation therapy the doctor has
B. Nonsteroid anti-inflammatory drugs E. Butadionum (phenylbutazone) administered a synthetic drug of nonsteroid structure
С. Antibiotics of Penicillin group
25. Specify a drug from the group of glucocorticoids, which eliminates stimulatory influence of oestrogens on
D. Ascorbic acid
which is poorly absorbed into the blood in application to tumoral growth. Specify this drug.
E. Sedatives
the skin. A. *Tamoxifen
20. Specify synthetic analogue of glucocorticoid A. *Synaflanum (fluoclnolone) B. Fosfestrol
hormones. B. Dexamethasone C. Rubomvcin
A. *Prednisolone C. Triamcinolone D. Diethylstiibestrol
В. Pituitrine D. Desoxycorticosterone E. Cisplatin
С Adrenaline E. Prednisolone
D. Cortisone 30. The woman of 28 years was admitted to the department
E. Testosterone 26. The doctor has administered an ointment containing of pathology of pregnancy due to threat of abortion. In her
glucocorticoid to the patient suffering from allergic anamnesis there are two events of premature birth. Specify
21. The patient suffering from severe form of diabetes dermatitis. The advantage of this drug is - it is not a drug of the yellow body hormone which should be
mellitus is netted by insulin. Now his condition requires absorbed into the blood from the skin. Specify this drug. administered in this case.
administering of antiinflammatory agent. Specify the drug A. *Flumethasone A. * Progesterone
which can demand correction of a dose of insulin? B. Dexamethasone B. Praegninum
A. * Prednisolone С. Becfomethasone С. Diazepam
26
D. Magnesium sulfate antihistamine drug' should be administered to a patient, A. *Diazofinum (Mebhydroline)
E. Vitamin E. taking into account his profession? B. Dirnedrolum (Diphenhydramine)
31. Cancer of mammary gland is diagnosed at the woman A. * Diazolinum C. Diprazinum (Promethazine)
of 6 years. What hormonal drug should be administered? B. Dimedrolum D. Tavegilum (Clemastine}
A. *Testosterone С. Diprazinum E. Suprastinum (Chloropyramine)
B. Synoestroium D. Suprastinum 2 5 . A 40 years old outpatient (driver) suffers from
C. Phenoboline E. Ketotifenum chronic conjunctivae, of allergic genesis. What
D. Progesterone 6. Which drug is the most preferable to local treatment of antihistamine drug is necessary to administer taking into
E. Insulin allergic dermatitis? account his occupation?
32. Indicate the drug which stimulates synthesis of A. *Hydrocortisone ointment A. *Diazolinum (Mebhydroline)
proteins, exerts positive influence on calcium and nitrogen B. Hippophea oil B. Diprazinum (Promethazine)
exchange and also promotes increase of appetite and body C. Furacilinum solution C. Suprastinum (Chloropyramine)
weight. D. Afethyisalicylate liniment D Dimedrolum (Diphenhydramine)
A. * Retabolilum (nandrolone decanoate) E. Ichthyolum ointment E. Ketotifenum
B. Prednisolone 21. Dimedrolum (diphenhydramine) is administered to a 27 It is necessary to administer antihistaminic agent to a
C. Dexamethasone patient with urticaria to reduce itching rashes on the skin. woman suffering from seasonal vasomotor rhinitis, who
D. Progesterone What mechanism provides its efficiency in this case? works as a dispatcher on the railway. Indicate the drug
E. Corticotropin A. * Competitive blockade of HI-histamine from this group which doesn’t possess sedative and
receptors hypnotic properties.
33. The doctor has administered Retabolilum (nandrolone
decanoate) to a female patient after consolidation of B. Inhibition of synthesis of histamine A.* Diazolinum (Mebhydroline)
fracture of a bone for acceleration of recovery, Indicate С. Suppression of release of histamine B. Dimedrolum (Diphenhydramine)
characteristic undesirable effect of this drug at women. D. Acceleration of histamine destruction C. Diprazinum (Promethazine)
E. Blockade of H2-his famine receptors D. Suprastinum (Chtoropyraine)
A. * Masculinization
E. Tavegilum (Clemastine)
B. Decrease of body weight 22. What drug should be administered to the patient who
С. Catabolic effect suffers from rashes due to allergic reaction accompanied 28. A girl was treated with antibiotic from the group of
D. Feminization by reddening, edema, and strong itch of skin which semisynthetic Penicillins due to acute bronchitis. On the
E. Asthenia causes sleeplessness? 3rd day of treatment allergic dermatosis has developed.
A. *Dimedrolum (diphenhydramine) Indicate an antiallergic drug which should be
34. In a woman, 33 years old, after long-term
B. Nitrazepamum administered to the patient.
pharmacotherapy of chronic polyarthritis arterial
C. Chlorall hydrate A. * Suprastinum (Chloropyramine)
hypertension, redistribution of fatty tissue, menstrual
D. Natrii oxybutyrate (oxybate sodium) B. Levamisole
disorders were observed. What drug did the patient take?
E. Phenobarbitalum С. Aspirin (Acetylsalicylic acid)
A. *Prednisolonum
D. Biseptol (Co-trimoxazole)
B. Indomethacinum 23. The patient with allergic rhinitis s taken antihistamine E. Mefenamic acid
C. Butadionum drug PO. In an hour the patient felt dryness in mouth,
D. Synaphlanum retardation and sleepiness. Indicate this drug. 29. Allergic dermatitis has been diagnosed at the patient
E. Beclomethasonum of 43. The doctor has administered to him complex
A. *Dimedrolum (Diphenhydramine)
therapy including the blocker of HI-histamine receptors.
B. Diazolinum (Mebhydrolin)
Indicate this drug.
ANTIALLERGIC AND IMMUNOTROPIC C. Phenobarbitalum
A. *Diprazinum (promethazine)
AGENTS D. Diazepamum
B. Cromolin sodium (cromoglicic acid)
E. Paracetamolum
C. Prednisolone
1. A 40 year-old patient working as a driver suffers from D. Adrenaline
24. Indicate "day time" antihistamine agent (with the least
chronic conjunctivitis of an allergic genesis. Which E. Hydrocortisone
expressed sedative and hypnotic activity).
27
30. Indicate the group of antiallergic agents which 38. A doctor administered chromoglicic acid to the patient during chemotherapy of malignant tumour. What drug
loratadine belongs to. suffering from bronchial asthma to prevent attacks. should be administered in this state?
A. * Blockers of histamine receptors Indicate the principle of action of this drug. A. * Thymalinum
B. Membrane stabilizers A. *Stabilization of membranes of mast cells B. Cyancobalamine
C. Antagonists of leucotriene receptors B. Binding of free histamine C. Iron preparations
D. Glucocorticoids C. Decrease of concentrations of immunoglobulins D. Prednisolone
E. Blockers of serotonine receptors D. Inactivation of histamine E. Acetylsalicylic acid
E. Blockade of histamine receptors
32. The patient of 35 years, suffering from bronchial 44. A drug from the group of immunostimulant which is
asthma, is hospitalized in a state of anaphylactic shock. 39. A 45 years old patient suffers horn seasonal allergic an analogue of natural biogenic substrate and almost
What of the numbered drugs is necessary to introduce rhinitis related lo blooming of ragweed. What agent doesn’t exert side effects is administered to a 4 years old
first of all as first aid? should be used for prevention of this disease? child. Indicate this drug.
A. *Adrenaline A. *Ketotifenum A. * Interferon
B. Dimedrolum (diphenhydramine) В. Phencarolum (quifenadine) B. Prodigiosanuin
С. Chromoglicic acid С. Diazolinum (mebhydroline) С. Thymalinum
D. Salbutamol D. Tavegilum (clemastine) D. Pyrogenatum
E. Ephedrine E. Dimedrolum (diphenhydramine) E. Levamisole
34. Anaphylactic shock was developed at the patient after 40. The patient took Levomycetinuin (chloramphenicole) 45. Indicate the immunostimulant drug which is also well
intracutaneous test on sensitivity to Penicillin. The doctor for a long time without doctor’s permission. Now the known as anthelmintic agent.
had administered a drug which eliminated bronchospasm patient’s examination revealed leucopnia. What drug A. * Levamisole
and arterial hypotension, indicate this drug. should be administered for stimulation of leucopoiesis. B. Interferon
A. * Adrenaline A.* Pentoxytum С. Methyluracil
B. Noradrenaline В. Methotrexate D. Pyrantelum
С. Mesatonum (Phenylephine) C. Mercaptopurine E. Natrium nucleinate
D. Atropine D. Cyancobalamine
46. A patient suffering from chronic generalized
E. Salbutamol E. Prednisolone
parodontics requires the treatment with immunostimulant
36. Indicate the drug which is the most expedient for using 41. The patient with chronic infection disease requires agent which possess anthelmintic activity. Indicate this
for topical treatment of allergic dermatitises? treatment with specific immunostimulant agent. Indicate drug.
A. *Hydrocortisone ointment this drug. A. *Levamisole
B. Solution of furacilinum (nitcofural) A. *Pentoxylum B. Seeds of pumpkin
C. Liniment of n>ethylsalicylate B. Methotrexate С. Pyrantelum
D. Ointment of ichthyolum (ichthammot) C. Mercaptopurine D. Chloxile
E. Sea buckthorn oil (Oleum Hippophae) D. Actinomycin E. Piperazine adipate
37. The patient with chronic relapsing dermatitis of E. Cyclophosphanum
47. Indicate the drug which significantly increases body
allergic genesis requires treatment by a glucocorticoid 42. 48 years old woman who is from exacerbation of temperature.
agent. Indicate the drug from this group which exerts only chronic pneumonia requires treatment with stimulant A. *Pyrogenatum
local action on skin and does not cause systemic side- agent. Indicate this drug. B. Acetylsalicylic acid
effects. A. *Thymalinum C. Methyluracil
A. * Synaflanum (fluocinolone) B. Sulfocamphocainum D. Aminazine (Chlorpromiazine)
B. Prednisolone C. Biseptol (co-trimoxazole) E. Molgramostim
C. Hydrocortisone D. Dimedrolum (diphenhydramine)
48. A 45-year-old woman suffers from allergic seasonal
D. Dexamethasone E. Levamisole
coryza caused by Ambrosia blossoming. What drug from
E. Triamcinolone 43. Indexes of immune response are worsened in a patient the group of stabilizers of mast cells can be used for
28
prevention of the disease? A *Loratadine B. Increase of myocardium excitability
A. *Ketotifen B Noradrenaline hydrotartrate C. Decrease of myocardium automatism
В. Phencarol C Propanolol D. Decrease of myocardium conductivity
C. Tavegil D Ambroxol E. Increase of diuresis and elimination of edemas
D. Dimedrol E Losartan
8. Indicate the mechanism of cardiotonic
E. Diazoline
action glycosides
49. A 12 year old child has intolerance to some foodstuffs.
CARDIAC GLYCOSIDES AND OTHER A. *Inhibition of Na-K-ATPase of cardiomyocyte
Their consumption causes an allergic reaction in form of
CARDIOTONIC DRUGS membranes
itching skin eruptions. What antihistaminic drug should be
admistered so that the child could attend school? B. Excitation of B-adrenoceptors of myocardium
1. A patient with pulmonary edema caused by acute left
A *Loratadine ventricular insufficiency patient was treated with cardiac С Activation of calcium channels of cardiomyocyte
B Dimedrol membranes
glycoside. In 10-15 min, his condition improved and
C Diclofenac maximal effect - was in 1-1,5 hours, after that the action D Oppression of phosphodiesterase of cardiomyocytes
D Aminophylline E. Activation of potassium channels of cardiomyocytes
gradually decreased. What drug has been injected?
E Ephedrine A. * Strophanthin 9. Indicate the group of drugs which is the most expedient
50. A woman works as railway traffic controller. She В. Digoxin for treatment of ciliary arrhythmia?
suffers from seasonal vasomotor rhinitis and gets treatment C. Celanidum A. *Cardiac glycosides
in the outpatient setting. She was prescribed an D. Digltoxin B. M -cholinoblockers
antihistamine that has no effect upon central nervous E. Adonisidum C. Na-channels blockers
system. What drug is it? D. Beta-adrenomimetics
2. A patient had been suffering from heart insufficiency
A *Loratadine E. Alfa-adrenoblockers
was treated with digoxinum. He took diuretic
B Dimedrol
Dichlothiazidum without the doctor's knowledge and 10. Indicate the group of drugs, overdosage of which is
C Promethazine
after 2 days he felt worse and was obliged to address this accompanied by following signs: nausea, vomiting,
D Suprastin
matter to the doctor. Doctor administered several drugs to diarrhea, infringement of heart activity (extrasystoles,
E Tavegil
improve patient's state and among these drugs was delay of atrioventricular conductivity), headache, vision
51. A 45-year-old woman suffers from allergic seasonal
Unithiolum. Indicate the mechanism of action of this impairment (xanthopsia, diplopia).
coryza caused by the ambrosia blossoming. What
agent A. * Cardiac glycosides
medicine from the stabilizer of the adipose cells group can
be used for prevention of this disease? A. *It has got free sulfhydril groups which bind to the B. Organic nitrates
A *Ketotifen molecules of digoxinum С Ca-channels blockers
B Diazoline B. Promotes excretion of calcium ions from the organism D. Beta-adrenoblockers
C Phencarol С. Promotes retention of potassium ions in the organism E. Angiotensin converting enzyme inhibitors
D Tavegyl D. Decrease oxygen demand of the myocardium 11. Why do strophanthin and corglycon possess fast
E Dimedrol E. Creates complexes with calcium ions action after the introduction into the organism?
52. A doctor administered a patient with allergic dermatitis
3. Specify the cardiac glycoside which posseses the A. * They have low affinity to plasma proteins
a H1-histamine blocker as a part of complex treatment.
fastest onset of the action. B. They have high molecular weight
Name this medication:
A. *Strophanthin С. They have low molecular weioht
A *Loratadine
В. Celanidum (lanatoside) D. They have high affinity to plasma proteins
B Cromolyn sodium
С. Digitoxin E. They have short half-life period
C Prednisolone
D. Digoxinum 12. A patient suffers from chronic left-ventricular
D Adrenaline
E. Adonisidum insufficiency. What medication should be administered?
E Hydrocortisone
A *Digoxin
53. A student came to see a doctor and asked to administer 4. What effect of cardiac glycosides is of greatest
B Bemegride
him a drug for treatment of allergic rhinitis that occurs in importance?
C Etimizole
the period of linden flowering. What drug may be used? A. *Increase of myocardium contractility
29
D Vinpocetine С. Local anaesthetics B. Novocainamide
E Pyracetam D. Beta-adrenoblockers C. Nitroglycerine
13. A patient that entered the admission office had the E. Potassium containing remedies D. Stropnanthin
following signs of acute cardiac insufficiency: paleness, E. Anaplilinum
2. A patient has lengthening of P-Q interval on the
acrocyanosis, frequent shallow respiration. What drug is
electrocardiogram under the treatment with an 7. Indicate the mechanism of anti arrhythmic action of
indicated in this case?
antiarrhythmic drug. What agent could cause it? quinidine sulphate
A *Corglycon
B Digitoxin A. *Atenolol A. *Blockade of sodium channels of the cardiomyocyte
C Cordiamin B. Prazosin membranes
D Nitroglycerine С. Atropine B. Blockade of calcium channels of the cardiomyocyte
E Adrenaline hydrochloride D. Lidocaine membranes
14. Patient complaines of weakness, dyspnea, low E. Plathyphyllin С. Blockade of β-adrenoceptors of the myocardium
extremities oedema. Diagnosis: chronic cardiac 3. It is necessary to appoint an anti-arrhythmic agent to a D. Blockade о α- and β-adrenoceptors of the
insufficiency. What medicine should be prescribed first of patient with ciliary arrhythmia accompanying by bronchial myocardium
all? asthma. What drug from listed below is contraindicated to E. Blockade of M-cholinoreceptors of the myocardium
A *Digitoxin this patient? 8. Specify the mechanism of anti arrhythmical action of
B Caffeine A. *Anaprilinum (propranolol) verapamil
C Papaverine B. Verapamil A. *Blockade of calcium channels of the cardiomyocyte
D Propranolol С. Aimalin membranes
E Raunatin D. Digoxine B. Blockade of sodium channels of the cardiomyocyte
15. A patient suffering from chronic cardiac insufficiency E. Novocainamidum (procainamide) membranes
was recommended to undergo a prophylactic course of C. Blockade or β-adrenoceptors of the myocardium
treatment with a cardiological drug from the group of 4. Indicate the mechanism of action of Verapamil
A * Blockade of calcium channels D Blockade of α- and β-adrenoceptors of the
cardiac glycosides that is to be taken enterally. What drug myocardium
was recommended? B. Inhibition of Na-K-ATPase
E. Blockade of М-сholinoreceptors of the myocardium
A *Digoxin С Activation of beta-adrenoceptors
B Strophanthine D. Activation of M-cholinoceptors
C Corglycon ANTIANGINAL DRUGS
E. Blockade of beta-adrenoceptors
D Cordiamin
5. A patient suffers from allergic reaction to iodine.
E Cordarone 1. Determine an antianginal agent by its pharmacological
Indicate an antiarrhythmic agent, which is absolutely
16. A 50 y.o. patient with chronic cardiac insufficiency and effect. It insignificantly improves coronary circulation
contraindicated to him.
tachyarrythmia was prescribed a cardiotonic drug. What (especially in subendocardial layers), but the main cause of
drug was prescribed? A. *Amiodarone
elimination of stenocardia is dilation of peripheral veins
A *Digoxin В. Verapamil
and arteries, that leads to decrease of cardiac work and
B Dopamine С. Novocainamidum (procainamide)
myocardial oxygen demand. Besides, it oppresses the
C Dobutamine D. Ornidum (bretytium)
central links of cardiostimulating reflexes.
D Amyodarone E. Quinidine sulfate
A. * Nitroglycerine
E Mildronate 6. In a clinic, the patient complained of unpleasant B. Phenihydinum
sensations in the heart region, and attacks of weakness С Amiodaronum
ANTIARRHYTHMIC AGENTS. and loss of consciousness. Inspection of the patient's D. Validolum
electrocardiogram had revealed the presence of II degree E. Anaplilinum
1. Indicate the group of drugs which is used for
atrioventricular block. Specify a drug which should be 2. A patient who had been suffering from stenocardia
treatment of atrioventricular blockade
used in this situation. accompanied by cardiac arrhythmia (paroxysmal
A. * M-cholinoblockers
A. * Isadrinum tachycardia) and arterial hypertension was admitted to the
B. Ca-channels blockers
30
hospital. Specify antianginal drug and the group which it A. *Organic nitrates D. Mildronate
belongs to, which should be administered taking into B. Beta-adrenoblockers E. Verapamil
account the patient's diseases. С. Blockators of calcium channels 12. A 50 years old patient has suffered from angina
A. *β-adrenoblocker - anaprilinum D. Activators of potassium channels pectoris for several months. As a rule he has successfully
B. Organic nitrate - nitroglycerinum E. Myotropic coronary dilators used a tablet of validolum during the attack but last 2
C. Myotropic spasmolytic agent – No-spa 8. After sublingual introduction of nitroglycerine its weeks this remedy hasn’t been effective. What drug should
D. Calcium antagonist - nifedipin maximal concentration in blood is developed in: be administered to the patient for elimination of the attack?
E. Potassium channel activator nicorandil A. * Nitroglycerine
A. * 4-5minutes
3. Indicate the nitroglycerin's drug with prolonged action B. 15minutes B. Nifedipine
A. *Sustac С. 1 minute С Verapamil
B. Validolum D. 30 minutes D. Anaprilinum (propranolol)
С. Amylnitrite E. 1 hour E. Isosorbide mononitrate
D. Natrium nitroprussid 9. Why are the tablets of nitroglycerine introduced 13. The calcium channels of cardiomyocytes have been
E. Dipiridamolum sublingually only? blocked on an isolated rabbit's heart. What changes in the
4. Indicate the drug which exerts antianginal action A. *The substance is being considerably destroyed heart's activity can happen as a result?
because of decrease of oxygen demand and increase of during its first passage through the liver A. *Decrease rate and force of heart beat
oxygen delivery to the myocardium. B. The substance is badly absorbed in the B. Heart stops in systole
A. *Nitroglycerinum gastrointestinal tract C. Decrease of heart beat rate
B. Anaprilinum С The 'substance operates reflexly from oral cavity D. Decrease force of the contraction
C. Dipiridamolum receptors E. Heart stops in diastole
D. Talinololum D. The substance is destroyed under the action of gastric 14. A patient with coronary artery disease was admitted to
E. Carbocromenum Juice the cardiological department. For stenocardia prevention a
E. It causes less side-effects in such way of introduction drug from the group of beta-adrenoceptor blockers was
5. Indicate the state which requires administration of administered. What drug is it?
nitroglycerine 10. Determine an antianginal agent according to its A *Metoprolol
A. *An attack of stenocardia pharmacological effects: dilating coronary arteries it B Atropine sulfate
B. Acute cardiac failure increases myocardial blood supply, dilating peripheral C Morphine hydrochloride
С Hypertensive crisis veins it decreases myocardial preload, dilating perypheral D Oxytocin
D. Chronic cardiac insufficiency arteries it decrease myocardial postload, besides it E Furosemide
E. Endarteritis obliterans oppresses the central links of coronaroconstrictive and 15. After a tooth extraction a patient felt persistent pain
cardiostimulatory reflexes: behind his breast bone. After sublingual intake of an
6. Indicate the mechanism of action of nitroglycerine A. *Nitroglycerine antianginal drug the pain behind the breast bone
A. * Release of NO groups which activate guanylyl B. Fenigidinum disappeared, but the patient complained of headache and
cyclase С. Amiodarone dizziness. What drug are these properties typical
B. Blockade of calcium channels D. Validolum for?
C. Blockade of adenosine receptors E. Anaprilinum A *Nitroglycerin
D. Activation ol adenylyl cyclase B Propranolol
E. Inhibition of phosphodiesterase 11. Introduction of an antianginal drug to a patient with
stenocardia caused improvement of patient's state and also C Metoprolol
7. А patient with ischemic heart disease complaints of arterial hypotension, tachycardia and throbbing headache. D Validol
worsening of his state that is caused by overdosage of Indicate this drug. E Verapamil
antianginal agent. What group of drugs can cause this state A patient who had myocardial infarction was administered
A. *Nitroglycerine
it it is known that concentration of methemoglobin in 75 mg of acetylsalicinic acid a day. What is the purpose of
B. Carbocromen
patient’s blood is increased? this administration?
С. Dipyridamole
31
A *Reduction of thrombocyte aggregation has been treating for 10 years. Indicate the group of drugs B. Captopril
B Inflammation reduction should be administered in this situation. С. Prazosinum
C Pain relief A. * Beta-adrenoblockers D. Nifedipine
D Temperature reduction B. Drugs of nitroglycerine E. Dichlothiazidum
E Coronary vessel dilatation C. Alfa-adrenoblockers
6. A doctor has administered to a patient clonidine
16. A patient suffers from stenocardia and takes isosorbide D. Sympatholytics (clophelinum) for elimination of hypertensive crisis.
mononitrate. He was prescribed a complementary drug E. Ganglioblockers
What class of hypotensive drugs does the named agent
with disaggregating effect. What drug is it? 2. A 45 year old patient, who had been suffering from belong to?
A *Acetylsalicinic acid idiopathic hypertension, was treated by an
A * Central neurotropic
B Nitroglycerine antihypertensive drug. After 4 days his arterial pressure
В. Peripheral neurotropic
C Propranolol decreased, but he complained of sleepiness and
С. Diuretics
D Nifedipine psychological suppresion. With which drug was the
D. Drugs affecting the renin-angiotensin system
E Validol patient treated?
E. Myotropic (vasotropic) hypotensive agents
17. A patient suffering from coronary artery disease had A * Clophelinum
taken a certain drug many times a day in order to arrest B. Prazozinum 7. A patient with hypertensive disease caused by raised
stenocardia attacks. Overdose of this drug finally caused С. Captopril sympathoadrenal system activity requires administration
intoxication. Objectively: cyanotic skin and mucous of a drug reducing neurogenic tone of vessels. What is
D. Enalapril
membranes, dramatic fall in the arterial pressure, the most suitable agent to be administered?
E. Apressine
tachycardia, respiration inhibition. Blood has increased A. *Clophelinum
concentration of methemoglobin. The drug the patient had 3. A patient who had been suffering from hypertonic B. Losartane
taken relates to the following group: disease had been treated for a long time with the drug С. Verapamil
A *Organic nitrates from the group of Rauwolf ia alkaloids and began to D. Hydrochlorthiazide
B alpha-adrenoceptor blockers complain of heartburn, pain in the epigastrial area and E. Apressinum (hydralazine)
C Calcium channel blockers bad mood. Indicate the drug which caused these
D Adenosine drugs complications. 8. Hypertensic crisis characterized by sharp headache,
E Myotropic spamolytics A. *Reserpinum dizziness, hyperemia of face, pains in the region of heart,
18. Anapriline therapy caused positive effect in the В. Octadinum rapid pulse, arterial pressure of 220/110 mm Hg has
dynamic of the disease of a 44-year-old woman С. Clophelinum developed in a patient suffering from essential
suffering from stenocardia. What is the main mechanism D. Papaverinum hypertension during the visit to the dentist. What agent is
of the effect of this medicine? E. Dlbazolum it necessary to introduce to the patient?
A *Blockade of beta-adrenoreceptors and decrease 4. A patient who had been suffering from arterial A. *Clophelinum (clonidine)
myocardial requirements to the oxygen. hypertension had taken a hypotensive drug, but in an hour В. Pinlenum fpempidine)
B Decrease of oxidative exchange in myocardium due to his blood pressure increased and 2 hours after it С. Timolol
enzyme blockade of Krebs' cycle decreased. Indicate this antihypertensive agent. D. Moxonidine
C Decreased power inputs of myocardium due to reduced A *Octadimlm E. Anaprilinum fpropranolol)
loading B. Prazosinum 9. Indicate the antihypertensive agent which can cause
D Increased oxygen supply to the myocardium С. Captoprii such side-effects as dryness in the mouth, constipation and
E Decreased need in increasing of oxygen supply to the D. Anaprilinum retention of water in the organism
myocardium E. Nifedipinum A. * Clophelinum (clonidine)
5. A patient had been suffering from hypertonic disease B. Coraiaminum (nikethamide)
HYPO- AND HYPERTENSIVE DRUGS
accompanied by chronic bronchitis with asthmatical С. Verapamil
component. Indicate the drug which is contraindicated D. Anaprilinum fpropranolol)
1. The patient suffering from arterial hypertension with
due to it's action on the bronchi. E. Nifedipine
hyperkinetic type of circulation and the high contents of
rennin, accompanied by stenocardia and sinus tachycardia A *Anaprillinum 10. Stable arterial hypertension was arisen in the patient
32
who had been suffering from chronic glomerulonephritis. chose noradrenaline in order to prevent hypotension. В. Sydnophenum
Indicate the most effective group of drugs to treat this What is the action mechanism of this С. Coffetnum
patient. medication? D. Meridilum
A. *Angiotensin converting enzyme inhibitors A *Activation of alpha-1-adrenoreceptors
E. Aminalonum
B. Ganglion blockers B Activation of serotonin receptors
С. α-adrenoblockers C Activation of beta-adrenoreceptors 4. A patient admitted to a hospital with complaints of
D. Myotropic spasmolytics D Activation of dopamine receptors decrease of memory, feeling of "noise" in the head. The
E. Calcium antagonists E Block of M-cholinoreceptors diagnosis which had established after examination was
11. A patient with hypertensic crisis was admitted to the atherosclerosis of brain blood vesseles. Indicate the agent
cardiological department, he was injected intravenously which can be administered to the patient.
THE AGENTS USED IN INSUFFICIENCY OF
with an antihypertensive drug - salt of an alkaline-earth A. *Clofibratum
CEREBRAL CIRCULATION. HYPOLIPIDEMIC
metal. What drug was injected? B. Euphyllinum
AGENTS
A *Magnesium sulfate С Dibazolum
B Potassium chloride 1. The patient who had been treated with a vitamin drug D. Nimodipin
C Sodium hydrocarbonate for the prophylaxis of brain vessel constriction complained E. Minoxidilum
D Calcium lactate of unpleasant sensations: blushing of upper part of the 5. A 58 year old woman suffered from cerebral
E Benzohexamethonium body, vertigo, flushing of blood to the head. Which drug atherosclerosis. The complex therapy administered by the
12. A patient with II stage hypertension has been taking exerts this effect physician included vitamins E and C. Indicate the role of
one of hypotensive medications for the purpose of A. * Nicotinic acid these drugs in the treatment of atherosclerosis.
treatment. After a time arterial pressure decreased, but the В. Tocopherol acetate A. *Inhibition of lipids' peroxydation
patient started complaining of flaccidity, sleepiness, C. Riboflavin B. increase of release of the pituitary gonadotropic
indifference. A bit later he felt stomach pain. He was D. Thiamini bromidum hormones
diagnosed with ulcer. What hypotensive medication has E. Calcium pangamate С. Decrease of release of glucocorticoids in adrenal
the patient been taking? cortex
A *Reserpine 2. A patient suffering from atherosclerosis is treated with
lovastatinum 0,04 g PO before sleep. Why is this drug D. Activation of the antitoxic function of the liver
B Dibazole E. Improvement of coronary circulation
C Furosemide administered once a day and before sleep?
A. *Cholesterol is synthesized only at night 6. A patient was admitted to the clinic of nervous diseases
D Verapamil
B. Development of sleepiness in the action of the drug. with increased arterial pressure (220/130 mm Hg). It was
E Captopril
С. In the evening the drug is better absorbed. diagnosed that he had ischemia of the left hemisphere of
13. A 66 year old female patient got intravenous injection
D. The cataboiism of a cholesterol goes mainly at night. the brain as a result of vessel spasm and impairment of
of magnesium sulfate solution for the purpose of
E. The cholesterol is excreted from an organism mainly venous outflow. Choose the preparation from the listed
elimination of hypertensive crisis. But arterial pressure
at night ones which is preferable for improvement of brain blood
didn't go down and after repeated introduction of the same
supply and which removes hypercoagulation in an acute
preparation there appered sluggishness, slow response,
3. A patient admitted to the neurology department with period of the illness:
inhibition of consciousness and respiration. What
complaints of severe headache, nausea, vomiting, feeling A. * Xantinol nicotinate
preparation is antagonist of magnesium sulfate and can
of numbness and weakness of the right arm, disorders of B. Caffeine (coffeinum)
eliminate symptoms of its overdose?
speech. BP – 220/130 mm Hg. During 15 years he had С. Meridilum (methylpnenidate)
A Calcium chloride
been suffering from arterial hypertension. After the D. Aminalonum (gamma-aminobutyric acid)
B Potassium chloride
examination the following diagnosis was made: ischemia E. Sydnophenum (pheprosidine)
C Sodium chloride
D Activated carbon of the left hemisphere due to vascular spasm and
7. Indicate the main effect of Piracetam
E Potassium permanganate impairment of venous outflow. Specify the drug which is
A. * Improves memory and cognition
14. A 63 y.o. man with collapse symptoms was delivered the most preferable for the improvement of cerebral blood
B. Decreases the integrating processes in the brain
to the emergency hospital. A doctor supply due to decrease of hypercoagulation.
С. Slows down synthesis of GABA in the brain
A. *Xanthinoli nicotinas
33
D. Reduces resistance of the brain tissue to hypoxia D. Parmidinum 3. A patient had taken celanidum for long time due to
E. increases brain necessity in oxygen E. Probucol chronic heart failure. The physician administered to him
8. A patient was admitted to the neurological department dichlothiazidum to eliminate leg edemas. Which drug
13. A patient has the family hypercholesterolemia.
complaining of memory impairment and decrease of should be taken together with the diuretic to prevent
Indicate the drug which may be used due to ability to
intellectual capacity after the car crash head trauma. Offer hypokalemia?
inhibit the main enzyme of cholesterol synthesis?
the remedy for improvement of metabolism in the brain: A. *Kalii chloridum
A. * Lovastatinum
A. * Pyracetam (Nootropil) B. Calcii chloridum
B. Colestipol
B. Nifedipinum С. Unithiolum
С. Cholestyramine
С. Sydnocarbum D. Natrii sulfas
D. Nicotinic acid
D. Caffeine (coffeinum) E. Magnii sulfas
E. Probucol
E. Analginum (metamizole) 4. A patient has been treated for a long time with cardiac
15. Indicate the hypolipidemic agent which may be used
9. What class of lipoproteins is the most atherogenic? glycoside digoxinum in connection with congestive heart
in atherosclerosis of brain arteries
A. * Low density lipoproteins failure. Now the patient's state is stable, but there are
B. Chylomicrons A. *Lovastatinum remaining edemas on the legs and face. What diuretic
С. High density lipoproteins B. Cinnarizine should be taken to avoid side-effects caused by
D. Very low density lipoproteins С. Pyracetam simultaneous administration of cardiac glycosides and
E. Intermediate density lipoproteins D. Tocopherol acetate diuretics?
E. Ascorbinic acid
10. Indicate the principle of an antiatherosclerotic action A. *Spironolactonum
of Lovastati-num 16. Specify the principle of antihy-perlipidemic action of B. Oxodolinum
A. *Oppression of endogenous cholesterol synthesis in lovastatinum. C. Dlchlothiazidum
the liver A. *Inhibition of synthesis of endogenous cholesterol in D. Diacarbum
B. Inhibition of peroxide radicals formation the liver E. Cyclomethiazidum
С. Infringement of exogenous cholesterol absorption B. Impairment of creation of superoxide radicals 6. The usage of dichlotiazide, etacrinic acid and
D. Inhibition of lipolysis in fatty tissue С. Impairment of absorption of cholesterol in the furosemide did not cause marked diuretic effect in the
E. Prevention or penetration of atherogenic intestine patient with marked peripheral edemas. The aldosterone
lipoproteins in tunica intima of vessels D. Impairment of lipolysis in the fatty tissue level in the blood is increased. Indicate which medicine
11. A patient with atherosclerosis of vessels was treated E. Impairment of binding of atherogenous lipoproteins should be prescribed:
with one of the hypolipidemic drugs which reduces with endotheliocytes A. *Spironolactone
cholesterol synthesis due to inhibition of enzyme 3- B. Mannit
hydroxy-3-methylglutaryl-coenzyme A reductase. DIURETIC AGENTS. DRUGS USED IN GOUT. C. Amilorid
Indicate the drug D. Clopamid
A. * Lovastatinum 1.The diuretic agent in dosage 0,025 g 2 times a day had E. Urea
B. Cholestyramine been prescribed to the patient with the beginning stage of 7. A doctor administered Allopurinol to a 26-year-old
С. Clofibrate idiopathic hypertension. In 7-8 days, the arterial pressure man with the symptoms of gout. What pharmacological
D. Nicotinic acid had slightly decreased, but he began to complain of pain in action of Allopurinol ensures therapeutical effect?
E. Probucol the heart region, muscle weakness, and tremor. The A. *Inhibition of uric acid synthesis
analysis of blood has revealed hypokalemia. Which from B. Increase of uric acid excretion
12. Among special hypolipidemic agents the most the listed drugs may cause this side effect
effective ones are those which block synthesis of C. Inhibition of leucocyte migration into the joint
A. * Hydrochlorthiazide D. Analgesic effect
endogenous cholesterol in the liver. What drug from
B. Spironolactonum E. Antinflammatory effect
listed below has such mechanism of action?
С Triamterenum
A. *Lovastatinum D. Amilorid 8. Indicate the diuretic agent which should be used to treat
B. Clofibrate E. Mannit pulmonary edema
С. Cholestyramine
34
A. *Furosemide A *Furosemide 2. A 40 year old patient has been suffering from bronchial
B. Hydrochlorthiazide B Hydrochlorothiazide asthma, accompanied with cardiac arrythmia (tachycardia)
С. Triamteren C Chlorthalidone for 10 years. Indicate adrenomimetic which should be
D. Spironolactone D Cyclometazide administered for treatment taking into account
E. Acetazolamide (diacarbum) E Acetazolamide accompanied disease.
9. A patient takes digoxin for treatment of cardiac A. *Salbutamol
insufficiency. What diuretic may increase digoxin toxicity THE AGENTS AFFECTING THE MYOMETRIUM B. Adrenaline
due to the intensified excretion of K+ions? TONE AND CONTRACTILE ABILITY. С. Isadrinum
A *Hydrochlorothiazide D. Orciprenalin
1.A 45 year old woman had referred to her gynaecologist
B Spironolactone E. Ephedrine
with complaints of menorragia. The doctor had
C Panangine 3. Indicate broncholytic which should be administered to
administered ergometrinum, which helped the woman.
D Siliborum a patient suffering from bronchial asthma accompanied
Specify the main effect of the drug.
E Lisinopril with stenocardia.
A. * Produces spastic contraction of the myometrium
10. A patient with chronic cardiac insufficiency has been A *Salbutamol
B. Accelerates process of coagulation
treated with cardiotonic drugs and a thiazide diuretic, but B. Ephedrine
С Oppresses proliferation of the endometrium cells
in spite of it there are still edemata and risk of ascites. С. Isadrinum
D Produces vasoconstriction endotheliocytes
What medication should be prescribed to amplify diuretic D. Orciprenalin
E. Produces rhythmic contractions of the myometrium
effect of the applied drugs? E. Adrenaline
A *Spironolactone 2. 1 ml of an agent causing contraction of the myometrium
4. A child was born with asphyxia. What agent is it
B Furosemide was introduced intramuscularly to a woman after abortion.
necessary to introduce for stimulation of breath?
C Amyloride In few minutes she began to complain of headache. AP
А. Promedolum
D Clopamide -160l100 mm Hg. Earlier she had initial stage of arterial
B.*Aetlumlzole
E Manitole hypertension. Choose among the following drugs which is
C. Prazozin
11. The alternate usage of dichlotiazide, etacrin acid and preferred in this case, taking into account the woman had
D. Atropine
lasix didn't cause marked diuretic effect in patient with inclination to arterial hypertension
E. Proserinum
marked peripheral edema. Increased amount of aldosterone A. *Oxytocin
5. To specify an analeptic which possesses sedative
is in the blood. Indicate the medicine to be prescribed? B. Pituitrinum
activity and can be used as desensibilizing agent in
A *Spironolacton С. Serotoninl adlpinas
bronchial asthma
B Mannit D. Hyphitocinum
A. *Aethimizole
C Clopamid E. Mammophysinum
B. Camphor
D Urea С. Cordiaminum
E Amilorid D. Carbogen
12. Diuretic drug was prescribed to the patient with THE AGENTS AFFECTING FUNCTIONS OF THE
E. Dimedrolum
hypertension in the course of complex treatment. In a few RESPIRATORY SYSTEM
6. Indicate antitussive agent possessing properties of
days BP decreased but signs of hypokaliemia developed.
1. А patient had been suffering from chronic bronchitis, opioid І analgesics
What drug could cause such complications?
A *Lasix was treated with an expectorant drug. In a week the A.*Codeine
B Spironolactone symptoms of rhinitis, tearing, itching of the skin and В. Libexinum
C Clophelin rashes appeared. What agent may cause these side C. Tussuprex
D Triamterene effects? D. Glaucine
E Enalapril A. * Potassium iodide E. Falimlnt
13. A 65 year old female patient suffers from chronic renal B. Terpinhydratum 7. An antitussive agent (1 tablet 3 times a day) was
insufficiency accompanied by evident edemata caused by С Acetylcysteinum administered to a patient. Cough has decreased but the
chronic glomerulonephritis. What diuretic should be D. Infusum herbae Thermopsidis patient has started complaining of dizziness, general
administered for forced diuresis? E. Alatriihydrocarbonas weakness and arterial hypotension has been' revealed.
35
Indicate the drug. В. Bromhexinum E. Acetazolamide (diacarbum)
A. *Glaucine C. Trypsin
18. Indicate the drug used for elimination of pulmonary
В. Codeine D. Acetylcysteine
edema caused by systemic arterial hypertension.
С Libexinum E. Libexinum
(prenoxdiazine) 13. Mark the group of drugs used for elimination of A. * Pentaminum
D. Oxeladine bronchial asthma attacks B. Strophanthin
E. Faiimint С Bemegridum
A. *Beta-adrenomimetics
8. Indicate the drug oppressing a peripheral link of D. Cordiaminum
B. M-chotinomimetics
cough reflex E. Spiritus aethylicus
С. Sympatholytics
A. * Libexinum D. Beta-adrenoblockers 19. Indicate the drug used in pulmonary edema
В. Codeine phosphate E. M-chotinomimetics accompanying by formation of foam to decrease
С. Ethylmorphine hydrochloride 14. A patient with bronchial asthma was treated with the superficial tension of bubbles to turn foam into
D. Bromhexinum combined drug in tablets that caused insomnia, irritability, A. *Spiritus aethylicus
E. Atropine suifate headache and rise of arterial pressure. What agent could B. Pentaminum
9. Thes mechanism of expectorant' action of cause these side-effects? С. Strophanthin
Thermopsis herb infusion is: A. *Ephedrine D. Bemegridum
A. *It stimulates bronchial secretion reflexively B. Adrenaline E Cordiaminum (nikethamide)
irritating the stomach receptors C. Libexinum 20. A patient suffering from chronic bronchitis takes a
B. Directly stimulates peristalsis of bronchial smooth D. Euphyliinum synthetic mucolytic drug that facilitates the sputum
muscles E. Furosemide thinning. What drug is it?
С. It destroys proteins of sputum A *Acetylcysteine
D. Irritates bronchial glands during excretion that leads 15. A patient suffering from bronchial asthma was treated
with the drug which caused in several days insomnia and B Diazepam
to stimulation of their secretion C Heparin
E. It inhibits the cough center tachycardia. Indicate this drug.
A. * Ephedrine D Furosemide
10. Indicate an expectorant agent possessing the reflex type E Enalapril
B. Plathyphylline
of action 21. A 70-year-old man, who suffered from chronic
С. Adrenaline
A, * Thermopsis herb infusion bronchitis, was prescribed medicine for the cough -
D. Euphyllinum
B. Bromhexinum codeine. What is the mechanism of anticoughing effect?
(aminiphylline)
С. Acetylcysteine A *Central
E. Salbutamolum
D. Trypsine B Reflex
E. Potassium iodide 16. Indicate the mechanism broncholytic effect of C Competitive
11. A patient with acute bronchitis I suffers from adrenaline D Local effect
intolerable dry cough. What from enumerated below A. * Stimulation of beta2-adrenoceptors E Peripheral effect
agents 'can transform dry cough into wet cough? B. Stimulation of beta1 and beta2-adnenoceptors
A. *Thermopsis grass infusion С Blockade of beta2-adrenoreceptors
B. Codeine phosphate D. Stimulation of alfa1 and alfa2-adrenoceptors THE AGENTS INFLUENCING FUNCTIONS OF
С. Libexinum E. Blockade of N-cholinoceptors THE DIGESTIVE SYSTEM
D. Glaucine hydrochloride 17. Indicate the diuretic agent which should be used to
E. Falimint 1. The patient was admitted to the hospital with the
treat pulmonary edema diagnosis: peptic ulcer of the duodenum bulbus. Analysis
12. Indicate an expectorant agent that is an inorganic
A. *Furosemide of his gastric juice revealed increased acidity. Choose the
substance and is usually used orally as a solution,
B. Hydrochlorthiazide agent which decreases the secretory ability of gastric
rarely as an inhalation, ft exerts direct irritating action
С. Triamteren glands due to blockade of H2-histaminic receptors.
on bronchial glands.
D. Spironolactone A. *Panitidinum
A. *Kalium iodide
36
B. Extract of belladonna of the duodenal bulb was treated with Farmotidin which D. Legaion (silimar in)
C. Atropinum caused him to feel better. Indicate the mechanism of E. No-Spa (drotaverine)
D. Methacinum action of this agent.
11. Indicate the agent which may be used in an attack of
E. Platyphytlinum A. *Blockade of Н2-histaminic receptors biliary colic to relax smooth muscles?
2. A 25 year-old man, suffering from peptic ulcer of the B. Inhibition of gastrin release
A. *Platyphyllinum
stomach, has been treated with omeprasole. In 3 weeks С. Suppression of the function of the gastric mucousal
B. Paracetamolum
the ulcer was healed. What mechanism of action does this cells
С. Analginum (metamizole)
drug produce? D. Decrease of release of hydrochloric acid
D. Pentazocine
E. Decrease of pepsin release
A. * Blockade of H+-K+-ANP-ase (the proton pump) E. Morphine
B. Blockade of M-cholinoceptors 7. A patient suffering from chronic hypoacidic gastritis
12. A 40 years old patient was admitted to the hospital
С. Blockade of synthesis of Gastrin with remained secretory function requires administration
with the biliary colic attack. What agent should be
D. Blockade of H+-K+-ANP аsе of an agent which is physiological stimulant of the gastric
administered in this case?
E. Blockade of H1 histaminic receptors glands. Indicate this agent
A. *No-spa (drotaverine)
3. Patient who had been suffering from chronic gastritis A. *Carbonaceous mineral water
B. Almagel
was treated with an antacidic drug, after introduction of B. Pepsin
С Pancreatin
which he felt better however at the same time he С. Histamine
D. Contrycal (aprotinine)
experienced bloating of the stomach together with D. Diluted hydrochloric acid
E. Metoclopramide
eructation. Indicate the drug which might cause this side E. Natural gastric juice
effect. 8. A patient with essential hypertension was treated for a 13. Indicate the drug from the group of myotropic
A. *Natrii hydrocarbonas long period of time with preparations containing reserpin. spasmolytics which is suitable to eliminate pain in
В. Magnesu trisilicate During last 2-3 months he started to suffer from pains in intestinal colic
the region of stomach, heartburn and nausea. The A. *Papaverine
С. Magnesii oxydum
diagnosis of hyperacidic gastritis was made after the B. Neostigmine (proserinum)
D. Almagel C. Piridostigmine
clinical examination. Indicate the group of drugs which
E Aluminii hydroxydum possesses etiotropic curative action in this case. D. Pilocarpine
A. *M-cholinoblockers E. Prazosine
4. Indicate the drug to stimulate appetite, mechanism of
action of which is associated with irritation of the mucus B. Astringent agents 14. Indicate a cholagogue agent used for treatment of chronic
membrane of the oral cavity, that leads to reflex excitation С. Antiacidic agents cholecystitis
of the hunger center in the hypothalamus. D. Inhibitors of proton pump A. *Allocnolum
A. *Absinthium tincture E. H2-histamine receptors blockers B. Absinthium (sagebrush) tincture
В. Phepranonum 9. Indicate the remedy increasing bile secretion: С. Metoclopramide
C. Desopimonum A. *Oxaphenamidum D. Almagel
D. Fenfluramine B. Apomorphine E. Plathyphyllin
E. Insulin С. Cimetidine 15. Specify an agent from the group of hepatoprotectors
5. Indicate the drug which increases appetite due to D. Almagel which restores normal structure and function of
decrease of glucose concentration in the blood E. No-Spa (drotaverine) hepatocytes, used in different liver deseases.
A. *Insulin 10. Indicate the agent which stimulates contraction of gall A. * Essentiale
B. Mazindolum bladder smooth muscle and causes evacuation of bile into B. Tetracycline
С. Fenfluramine the intestine? С. Cholenzymum
D. Absinthium tincture D. Tocopherol acetate
A. *Magnesium sulfate in enteral introduction
E. Phepranonum E. Allocholum
B. Magnesium sulfate in parenteral introduction
6. A 32 year old patient had been suffering from the ulcer С. Dehydrocholic acid 16. Indicate the drug, which is used in chronic
37
pancreatitis, accompanied by enzymesT insufficiency, has a sensation of stomach swelling. Which of the complaints of eye discomfort, discharge of purulent
for improvement of digestion processes. following drugs might be the cause of such side effect? exudate, disorders of vision. Specify the antiseptic
A. *Festalum A *Sodium hydrocarbonate available for rinsing of the eyes.
B. Pepsin B Magnesium oxide A. *Silver nitrate
С Acidin-pepsinum C Magnesium trisilicate B. Lugol's solution
D. Natural gastric Juice D Aluminium hydrooxide С Potassium permanganate
E. Diluted nydrocnloric acid E Pepsin P. Salicylic acid
17. Why is contrycal (aprotinine) used in the case of 22. A liquidator of a breakdown at a nuclear power plant E. Ammonium solution
acute pancreatitis? who was irradiated complained about vomiting that occurs
all of a sudden. What medication should be prescribed? 4. In a patient with varicose dilation of veins the trophic
A. *It inactivates trypsin which causes autolysis of
A *Metoclopramide ulcer of the leg developed. The bacteriological
pancreas
B Reserpine examination of the ulcer discharge revealed
B. It opens Oddies sphincter
C Atropine Staphylococcus infection. For the local treatment of the
С. It reduces the activity of hyaluronidase
D Aeron ulcer an antiseptic in the form of ointment from the
D. It impairs secretion of trypsinogen
E De-Nol group of detergents was administered. Specify it.
E. It oppresses secretion of bile
A. *Ethonium
18. Indicate an anti-enzymatic agent inhibiting activity B. Brilliant green
of trypsin, kallikrein and fibrinolysis ANTISEPTICS AND DESINFECTANTS
С Furacil/inum
A. *Confrykal (aprotinine) 1. The patient addressed to the doctor in relation with D. Potassium permanganate
B. Cholenzymum trauma of the foot. The foot was bandaged with a dirty E. Ethacridini iactas
E. Pancreatin gauze bandage, impregnated with purulent discharges.
5. Which acid possesses the properties of an antiseptic?
D. Pancreozymin Attempt to take off a bandage for survey and processings
A. *Boric acid
E. Festal of a wound invoked an acute pain as the bandage had
B. Nicotinic acid
19. A 37-year-old man was admitted to the surgical stuck to wound surface. Choose an antiseptic which will
C. Folic acid
department with the symptoms of pancreatitis: vomiting, facilitate taking off of a bandage and will mechanically
D. Ascorbic acid
diarrhea, bradycardia, hypotension, weakness, clear a wound of mud and pus.
E. Dehydrocholic acid
dehydration. What medicine should be used first of all? A. * Hydrogen peroxide
В. Aethacridinum lactate 6. Specify the antiseptic which is used for disinfection of
A. *Contrycal operation field and surgeon's hands.
С. Aethonium
B. Etaperazine
P. Potassium permanganate A. *70% solution of ethyl alcohol
C. No-spa
E. Furacilinum B. Furacilinum
D. Platyphylline
C. Ethonium
E. Ephedrine 2. The patient addressed to the doctor with complaints of
D. Potassium permanganate
20. A 37-year-old man was admitted to the surgical pustular pimples on the skin of the face. In
E. 95% solution of ethyl alcohol
department with symptoms of acute pancreatitis: vomiting, bacteriological analysis of contents of pustules
diarrhea, bradycardia, hypotention, weakness, dehydration staphylococcus aureus was found and the diagnosis of 7. Determine the following drug: it contains an halogen,
of the organism. What medicine should be used first of all? staphylococcal pyodermia was given. Choose the most exerts antimicrobial and deodorizing action, is used for
A *Contrycal efficient drug from the listed antiseptics for local use in disinfection of non-metal instruments, as an antiseptic -
B No-spa pustular pimples. for processing of hands
C Platyphylline A.* Brilliant green A.*Chloraminum
D Etaperazine B. Ethyl alcohol B. Hydregen peroxide
E Ephedrine С. Chlorhexidinum C. Formaldehyde
21. A patient suffering from chronic hyperacidic gastritis P. Potassium permanganate D. Phenol
takes an antacid drug for heartburn elimination. After its E. Aethacridinum lactate E. Resorcinum
ingestion the patient feels better but at the same time he 8. A patient was admitted into the emergency
3. A patient addressed to г ophthalmologist with
38
department in relation with acute poisoning - by mistake E. Interaction with hydroxilic groups of microbes 17. For the preparation of a patient's burn skin surface a
he drank mercury dichloride solution. The patient enzymes certain medication was used. Its antiseptic action is
complained of severe pain in the oral cavity, along the provided by free oxygen that segregates in presence of
13. A doctor used 5% spirituous solution of iodine for
esophagus and in the epigastric area, hypersalivation, organic substances. Choose the right answer:
cleaning of operation field. Indicate its mechanism of
fatigue, tachycardia. Specify the agent which would A *Potassium permanganate
action:
neutralize the absorbed mercury binding to it. B Furacilin
A. *Interaction with amino groups of microbes 'proteins C Chlorhexidine
A. *5% solution of unithiolum
that disposes to their denaturation D Boric acid
intramuscularly
B. Dehydration protoplasm's proteins E Sodium bicarbonate
B. Methylene blue with 5% glucose solution
C. Bound to enzymes' sulfhydric groups
intravenously solution of sodium
D. Formation of albuminates
C. 4% solution of sodium carbonate intravenously
E. inhibition of dehydrogenase
D. 2% solution of sodium nitrite intravenously SULFONAMIDES AND OTHER SYNTHETIC
E. 2% solution of furosemide 14. A female suffers from varicose veins dilatation of ANTIBACTERIAL AGENTS
lower extremity which is complicated by ulceration on
9. All antiseptics possess ail following properties except:
ankle. The ulcer is accompanied by local hyperemia and 1. The doctor administered Sulfadimezinum in tablets to
A *Selective antimicrobic action itching around and discharges pus with staphylococci. An
B. Versatile antimicrobic action the patient with bacterial infection, and advised to take the
antiseptic ointment from the group of detergents was drug with alkaline mineral water. Indicate the purpose of
С. Bactericidal action administered for treatment. After treatment all of the
D. Highly toxic for human the given reference.
symptoms are diminished. Indicate the drug:
E. Are not introduced parenterally A. * For prophylaxis of crystallization of acetylated
A. * Ethonium derivants of the drug in renal tubules
10. Chloramine possesses all following effects, except: B. Furacilhium fnitrofuran) B. For prolongation of action
A. *Antiallergical С Viride nitens (brilliant green) С For reducing of the irritative action on the stomach
B. Deodorization D. Aethacrydine lactate D. For neutralization of HCl of a gastric juice
C. Antiseptic E. Potassium permanganate E. For shift of blood pH in the alkaline side
D. Spermicidal 15. Burned skin surface was treated with a certain
E. Fading preparation. Its antiseptic properties are provided by 2. A 37 year-old patient was admitted to an infectious
atomic oxygen that is formed in presence of organic diseases hospital with the diagnosis of dysentery. Indicate
11. Formaldehyde solution is used for disinfection of non- substances. What preparation was applied? the drug which should be appointed to the patient?
metallic surgical tools. Indicate the correct name of group A *Potassium permanganate A. * Ciprofloxacin
of formaldehyde: B Furacillin B. Erythromycin
A. * Aliphatic agents C Chlorhexidine bigluconate С. Oxacillinum
B. Aromatic agents D Alcoholic iodine solution D. Phenylsalicylate
C. Spirits E Sodium hydrocarbonate E. Imodium
D. Halogen-maintained agents 16. Patient with abscess of the cut wound applied to the
E. Detergents 3. Specify the sulfonamide drug which is poorly absorbed
traumatological department. Doctor for the cleaning of the
in the intestine and is used for the treatment of intestinal
wound from the pus washed it with 3% hydrogen
12. 70% solution of aethyl spirit was used by a surgeon for infections.
peroxide. Foam was absence. What caused the absents on
cleaning his hands before operation. Explain the A. *Phthalazolum
the drug activity?
mechanism of action of the antiseptic drug: B. Ethazolum
A *Inherited insufficiency of catalase
A. *Protein dehydration of microbes protoplasm B Low concentration $H_{2}O_{2}$ С. Sulfadimethoxinum
B. Blockade of sulfhydryl groups of enzymes C Inherited insufficiency phosphatdehydrogenase of D. Sulfadimezinum
C. Oxidation of organic components of microbes erythrocyte E. Sulfacylum-natrium
'protoplasm D Shallow wound 4. Specify the sulfonamide agent for the treatment of
D. Interaction with aminogroups of protoplasm proteins E Pus in the wound conjunctivitis.
of microbes
39
A. *Sulfacylum-natrium D. Nitroxolinum D. Aminoglycosides
B. Phthalazolum E. Furaginum E. Cephaiosporines
С. Urosulfanum
9. During examination in out-patient department a 13. After long-term treatment by antibiotics at the in-
D. Sulfadimezinum
physician identified pneumonia and- prescribed in-patient patient department a patient developed dyspeptic
E. Biseptoium
treatment by ampicillin and cefalexin. However, the syndrome. Investigation of stool revealed diminished
5.Specify the combined sulfonamide agent. patient started treatment at home with the same antibiotics, amount of Bifidobacterium and Bacillus coli. What is the
A. *Biseptolum dosage and timing prescribed by the doctor. Within three reason of present illness?
B. Ethazolum days the sick person felt better, fever and cough reduced. A. *Disbacteriosis
С. Sulfacylum-natrium The treatment was discontinued and the patient turned B. Enteric colibacillosis
D. Sulfadimethoxinum back to work. Next day he/she felt much worse, fever and C. Pseudomembranous enterocolitis
E. Streptocidum cough were developed again, that is why the patient had to D. Toxic action of the agents
be examined by the physician, indicate please which E. Acquisition of nosocomial infection
6. A patient visited a physician with complaints of painful principle of chemotherapy was disobeyed by the patient:
and frequent urination, pain in the lower part of the back. 14. Indicate an antibacterial agent from the group of
A. *Duration of treatment
After laboratory and bacteriologic examination of the fluoroquinolone:
B. Combined usage of agents
urine (it revealed gram-positive cocci, Proteus, acute A. *Ciprofloxacin
C. The earliest chemotherapy beginning
cystitis and urethritis were diagnosed. Specify the agent B. Nalidix acid
D. Effective agent choice based on clinical and
that should be administered taking into account the С. Nitroxolin
bacteriological diagnosis
localization of its action. D. Furosemide
E. Optimal selection of dosage, timing and
A. *Nitroxlinum E. Biceptol
introduction ways of agent
B. Ethazolum 15. Patient with pneumonia has intolerance to antibiotics.
C. Biseptoium 10. Patient with pneumonia was treated by injections of Which of the conbined sulfanilamide medicines should be
D. Sulfadimezinum antibiotic. Determine the type of chemotherapy: prescribed to the patient?
E. Furasolidonum A. *Causal treatment A *Biseptol
6. Specify the antimicrobial drug from the group of 8- B. Substitute treatment B Aethazol
oxiquinolirte derivatives. C. Preventive treatment C Natrium sulfacyl
A. *Nitroxolinum D. Symptomatic treatment D Streptocid
B. Biseptoium E. Pathogenic treatment E Sulfadimethoxine
C. Nalidixic acid 16. A 7 year old child is ill with bronchitis. It is necessary
11. Duration of sulfonamide agents' activity depends on:
D. Ciprofloxacinum to administer him an antibacterial drug. What drug of
A. *Affiliate activity with proteins of blood or/and fluoroquinolone group is contra-indicated at this age?
E. Furaginum reabsorption in renal canaliculi A *Cyprofloxacin
7. Specify the antimicrobial drug from the group of B. Aptitude of enterohepatic circulation B Ampicillin
nitrofurans. С. Speed of absorption in GIT C Amoxicillin
A. *Furaginum D. Level and speed of metabolic biotransformation in D Sulfadimethoxine
B. Biseptoium liver E Ampiox
С Nalidixic acid E. All named above 17. Gonorrhoea was revealed in the patient on
D. Nitroxolinum 12. After antimicrobial treatment of pneumonia within 8 bacterioscopy of the smear from urethra. Taking into
E. Ciprofloxacinum days patient developed painful urination, pain in kidneys account that medecines for gonorrhoea are
8. Specify the antimicrobal drug from the group of area, the urine has brown. Indicate the drugs group that fluorquinolones, patient should be prescribed:
fluoroquinolones. used in the case: A *Ciprofloxacin
A. *Sulfonlamide B Furazolidone
A. *Ciprofloxacinum
B. Lincomycin C Fluorouracil
B. Biseptoium
C. Β- lactam antibiotic D Urosulfan
С. Nalidixic acid
40
E Cefazoline B. Macrolides A. *Detergent activity
C. Tetracyclines B. Mechanism of action linked to infringement of
ANTIBIOTICS (I) D. Aminoglycosides microbe s membrane synthesis
1. A 60-year-old patient was admitted to the surgical E. Lincosamides C. Bactericidal activity
department because of infection caused by blue pus D. Distinguished from penicillines by higher
8. Specify the most typical side-effect of penicillins.
bacillus (Pseudomonas aeruginosa) which is sensative to persistence toward β-lactamase
A.*Allergic reactions
penicillin antibiotics. Indicate which of the given E. Distinguished from penicillines by spectrum of
B. Agranulocytosis
penicillins has marked activity to the Pseudomonas antimicrobial activity
C. Anemia
aeruginosa?
D. Decrease of audition 18. 56-years old male was admitted to a hospital with
A *Carbenicillin disodium
E. Hepatotoxic influence pneumonia. It is known he suffers from hay fever and
B Benzylpenicillin
9. Specify the antibiotic from the group of semisynthetic seasonal vasomotor rhinitis. What drug should be
C Phenoxymethylpenicillin
penicillins. administered in the case?
D Oxacillin
E Methicillin A. *Ampiciilinum A. *Cefazolin
2. A 19 year old woman suffers from primary syphilis. B. Phenoxymethylpenici/linum B. Benzylpenicillin
Doctor administered her complex therapy that includes C. Benzylpenicillinum natrium С. Bicillin
benzylpenicillin sodium salt. What is the mechanism of D. Benzylpenicillinum kalium D. Oxacillin
action of this drug? E. Benzylpenicillinum novocainum E. Ampiciliin
A *It blocks synthesis of peptidoglycan of microbal 19. 14-years old boy developed acute pneumonia in low
14. Indicate a drug group which oppresses synthesis of cell
membrane lobe of the right lung. The agent in sputum analysis was
membrane components:
B It blocks synthesis of cytoplasm proteins resistant to penicillin. Choose the drug for treatment in
A. *Penicillines
C It blocks thiol enzymes this case:
B. Tetracyclines
D It blocks RNA synthesis A. Gentamycin
C. Aminoglycosides
E It blocks DNA synthesis B. Laevomycetine
D. Lincosamides
3. A patient was delivered to the surgical department with E. Macrolides С. Streptomycin
anaerobic gangrene. Specify the antibiotic of first choice D. Tetracycline
15. Drug with β-lactam ring was prescribed to a patient E. *Cefazolin
for the treatment of this infection.
with streptococci gums inflammation. Indicate this drug:
A. *Benzylpenicillinum natrium 20. Determine drug by following: it oppresses of protein
В. Tetracyclinum A. *Benzylpenicin
B. Rifampicine synthesis by microbes ribosomes because of inhibition of
С. Clindamycinum peptidtranslocase, belongs to reserve macrolide, causes
D. Cefazolinum C. Erythromycin
D. Streptomycin sulfate side effects relatively seldom.
E. Chloramphenicol A. *Erythromycin
E. Laevomycetine
4. Specify the main antibiotic for the treatment of B. Sygmamycin
diphtheria. 16. A patient was admitted to a hospital with diagnosis: С. Tetraolean
A. *Erythromycinum gaseous gangrene. Drugs for its treatment are divided on D. Azithromycin
B. Laevomycetinum two groups: basic and reserve. Indicate the basic E. Tetracycline
С. Cefazolinum antibiotic:
A. *Benzylpenicillin natrium 23. A patient with diminished hearing has severe bacillary
D. Gentamycinum infection. Which drug group is contradicted to the patient?
E. Tetracyclinum B. Tetracycline
C. Laevomycetine A. *Aminoglycosides
7. Specifу the group of antibiotics whose mechanism of
D. Clindamycin B. Peniciltines
action is connected with inhibition of synthesis of bacterial
E. Cefazolin C. Cephalosporines
ceil wall.
17. Cephalosporines possess following D. Tetracyclines
A. *Penicillins E. Rifampicines
properties, except:
41
24. Patient with acute appendicitis, was admitted to a 26. An antibiotic with ability to penetrate to bones tissue B. Macrolides
surgical department. Appendectomy was performed. was prescribed to 30 years old patient with osteomyelitis. C. Penicillins
During ten days after operation patient received an After three weeks of using it the patient felt much better. D. Cephalosporins
antibiotic. After a while lowering of hearing were Determine the drug: E. Aminoglycosides
revealed. Indicate drug group with the same side effects: A. Lincomycin
6. An antibiotic was administered to a patient suffering
A. *Aminoglycosides B. Bicilline-3
from abdominal typhoid. Soon there was general
B. Tetracyclines C. Benzylpenicillin
improvement, but on the 2nd week after the treatment the
C. Polymyxines D. Polymixine M
patient had elevation of body temperature, signs of
D. Macrolides E. Ampicillin
tonsillitis, and rashes on mucous membranes of lips &
E. Penicillines 27. A patient started to complaint of worsening of audition nose. In laboratory examination of discharges, Candida
25. Determine the drug for treatment infections of bones after treatment with antibiotic because of purulent fungi were found. The blood analysis revealed leukopenia
that able to penetrate to bone tissue and bone marrow: complication after the surgical operation. Specify the and agranulocytosis. Which antibiotic could cause these
A *Lincomycin group of antibiotics which posses ototoxic activity. complications?
В. Benzylpenicillin A. * Aminoglycosides A. *Laevomycetinum
С. Bicitlin-3 В Penicillins B. Tetracyclinum
D. Gentamycin С. Tetracyclines С. Polymyxins
E. Synthomycinum D. Polymyxins D. Gentamycinum
29. A patient with diagnosed streptococcal E. Macrolids E. Cefazolinum
bronchopneumonia after treatment with an antibiotic 28. A patient with bacterial pneumonia was prescribed 10. Specify the group of antibiot ics whose mechanism of
suffers from allergic symptoms. Determine the drug: benzylpenicillin. What is the mechanism of its antibacterial action involves inhibition of protein synthesis by
A. *Benzylpenicillin-natrium (penicillin G sodium) effect? microorganisms.
B. Tetracycline A *Inhibition of synthesis of microorganism wall A. *Tetracyclines
C. Gentamycin B Inhibition of intracellular protein synthesis B. Penicillins
D. Laevomycetineum (chloramphenicol) C Abnormal permeability of cytoplasmic membrane С. Cephalosporins
E. Doxicycline D Inhibition of SH-groups of microorganism enzymes D. Monobactams
E Antagonism with p-amino-benzoic acid E. Polymyxins
30. Infectious agent determined by lab tests is known to be
sensitive to third generation cephalosporins. Choose the 11. In the treatment with wide-spectrum antibiotics some
drug for treatment: ANTIBIOTICS (II). ANTIFUNGAL AGENTS complicaions, including candidiasis may occur. Specify the
A. Cefazolin agent for the treatment of candidiasis.
B. Cefalexin 1. The patient with the diagnosis of cholera was A. *Ketoconazole
С Cefalotin admitted to the infection diseases hospital. Specify a В Amphotericinum B
D. *Cephtriaxone group of antibiotics of the first choice for treatment of C. Griseofulvinum
E. Cefaloridin this disease D. Gramicidinum
A. *Tetracyclines E. Undecinum
31. A patient with bacterial pneumonia was treated by the B. Aminoglycosides
erythromycin which acts on microbes by interaction with С. Penicillins 12. A patient with dermatomycosis took antifungal agent
their free 50S subunits of ribosomes. What process does D. Macrolids which was able to be accumulated within the cells
this drug block? E. Cephalosporines producing keratin (skin, nails, hairs), in several clays the
A. *Proteins' synthesis patient visited the physician complaining of headache,
B. RNA synthesis 5. A woman addressed to a dentist with complaints of teeth desorientation. Specify the appointed antibiotic.
C. DNA synthesis destruction in her little child. It was revealed that during
A. *Griseofulvinum
D. Lyposynthesis pregnancy the woman took antibiotics. Specify the group
B. Levorinum
E. Polysaccharides' synthesis of antibiotics that could cause these side-effects.
С. Amphotericinum В
A. *Tetracyclines D. Mycogeptinum
42
E. Nystatinum 32. An antibiotic for treatment of enteric fever was D Benzylpenicillin sodium salt
administered to a patient. Clinical recovery was achieved, E Biseptol
13. After long-term treatment with tetracyclinum a patient
but within 2 weeks the patient developed symptoms of 37. Patient was admitted to the infection unit with
was hospitalized in relation, with aphthous stomatitis.
quinsy, fever, rashes at mucous membranes of lips and diagnosis of bacterial dysentery. On laboratory studies it
During laboratory examination the Candida fungi were
nose. Blood test revealed diminished amount of WBC and was revealed that causative element is sensative to the
identified. Specify the agent available for the treatment of
granulocytopenia. Choose an antibiotic which can cause many antimicrobial medicines, but patient has anemia.
candidiasis.
these side effects: What medicine is contra-indicated to the patient?
A. *Nystatinum
A. Tetracycline A *Levomycetin
В. Furazolidonum
B. *Laevomycetinum (chloramphenicol) B Phthalazol
С. Griseofuivinum
С. Роlymyxine М sulfate C Furazolidone
D. Amicazolum
D. Cefazoline D Enteroseptol
E. Cefalexinum
E. Gentamycine E Ampicillin
21. Determine drug with wide spectrum of antibiotic 38. A patient suffers from severe postoperative
activity, a basic antibiotic agent of treatment enteric fever 33. Tetracycline was administered PO for treatment of pseudomonadous infection. What of the following
and other salmonellosises and possesses following side acute purulent sinusitis. What antimycotic drug should be antibiotics should be administered in this case?
effects: oppresses of bone marrow activity, disbacteriosis administered to a patient to prevent candidiasis? A *Amicacin sulfate
and dyspeptic disorders: A. Griseofulvin B Benzylpenicillin
A. *Laevomycetine B. Levamisole C Cephazolin
B. Phthalazolum C. Furazolidone D Erythromycin
С. Benzylpenicillin natrium D. Ciprofloxacin E Doxycycline
D. Neomycin sulfate E. *Nystatin
E. Tetracycline 34. After taking tetracycline for a long period of time, ANTITUBERCUFOUS, ANTIVIRAL AND
patient developed candidiasis of mucous membranes of ANTISYPHILITIC AGENTS.
22. Which drug is used for treatment of enteric fever?
A. *Laevomycetine mouth. Which drug should be used for treatment? 1. The patient who had been suffering from tuberculosis
B. Ampicillin A. *Nystatin was treated with Isoniazidum. After a while the patient
C .Cefalexin B. Griseofulvin began to complaint of muscle weekness, decrease of skin
D. Benzylpenicillin С. Nitrofungine sensitivity, impairment of vision and motor
E. Erythromycin D. Nitroxoffne discoordination. Indicate the vitamin's drug which should
E. Furadon/ne be administered to eliminate the specified phenomena?
27. Deteirmine drug for treatment of candidiasis: 35. A patient underwent appendectomy. In the
A. *Nystatin A. * Pyridoxin (B6)
postoperative period he has been taking an antibiotic. The B. Retinol (A)
B. Kanamycin patient complains about hearing impairment and vestibular
C. Tetracycline С. Ergocalciterol (D)
disorders. What group of antibiotics has such by-effects? D. Cyanocobatamin (B12)
D. Erythromycin A *Aminoglycosides
E. Benzylpenicillin E. Ascorbinic acid (C)
B Penicillins
28. Considerable number of Candida albicans was C Tetracyclines 2. Indicate the drug which is used for intranasal dropping
revealed on cytological investigation of smear of 25 D Macrolides with the purpose of prophylaxis of infuenza.
years-old woman with exacerbation of chronicle vaginitis. E Cephalosporins A. * Interferon
Which drug should be prescribed? B. Remantadinum
A. *Nystatin 36. A patient has herpetic rash. What medication should be С. Ampicillinum
B. Amphotericine administered? D. Aciclovir
C. Miconazole A *Acyclovir E. Paracetamolum
D. Clotrimazole B Gentamycin
C Clotrimazole 3. Specify the antibiotic available for the treatment of
E. Metronidazole
43
tuberculosis. D. Anaiginum B. Isoniazid
A.*Rifampicinum E. Paracetamolum С. Streptomycin
B. Tetracyclinum D. Ethionamide
9. In the newborn department of a hospital there was
С. Ampici/linum E. PAS
sudden increase of acute respiratory disease caused by
D. Erythromycinum venous types of viruses. To prevent spread of the infection 14. Patient with leprosies developed hypopigmentic rash
E. Lincomycinum it was recommended to use human leukocytic interferons. with absence of perception in its location. An antibiotic
4. Specify the antituberculous agent which inhibits Specify the available way of introduction in this case. that is the basic antituberculous agent was prescribed.
synthesis of mycolic acids by Mycobacterium tuberculosis. A. *Intranasal Indicate this drug:
A. *Isoniazidum B. Subcutaneous. A. *Rifampicine
B. Ethambutolum С Intramuscular B. Amoxicillin
C. Streptomycinum D. Peroral С. Erythromycin
D. Cycloserinum E. Inhaled. D. Nitroxoli
E. PAS E. Cefazolin
10. A woman 25 years old was hospitalized for treatment
5. Indicate the most effective synthetic antituberculous of syphilis. Specify one of the main antibiotics for 15. After treatment of patient suffering from tuberculosis,
drug. treatment of this disease. his vision worsened rapidly, visual fields were narrowed.
A. Kanamycinum A. *Benzylpenicillinum natrium Determine the drug which caused these side effects:
B. Streptomycinum В Erythromycinum A *Ethambutol
С. Rifampicinum C. Tetracyclinum B. Isoniazid
D. PAS D. Lincomycinum C. Kanamycin sulfate
E. *Isoniazidum E. Vancomycinum D. Ethionamide
11. A 35-year-old man under the treatment for pulmonary E. Rifampicine
6. A patient, 60 years old had been treated for tuberculosis
for a long time. Recently he began to omplain of decrease tuberculosis suffers acute pain of the right big toe, 16. 60-Years old male, with diagnosed tuberculosis long
in audition, which drug should be contraindicated? accompanied by swelling and slight fever. The gouty time ago, timely receives antituberculous treatment. He
A. *Streptomycinum arthritis was diagnosed and high serum uric acid level was developed neuritis of nervous facials. Which drug can
B. Ftivazidum found. Which of the following antituberculos drug is cause this side effect and should be accepted from the
С Ethambutolum known for causing high uric acid levels? therapy:
D. Isoniazidum A. *Pyrasinamide A *Ethambutol
E. Rifampicinum B. Cycloserine B. Isoniazid
С. Rifampicine С. Rifampicine
7. Drugs of which group must be administered first of all D. Thioacetazone D. Streptomycin
to a girl 1.5 years old in relation with acute herpetic, E. Aminosalicylic acid E. PAS
stomatitis during rash period?
A. *Antiviral agents 12. Patient has inherited of acetyl-transferase 17. After treatment by antituberculous drugs during three
B. Antiallergic agents insufficiency. Which drugs can cause severe intoxication month, a patient developed daltonism, reduced ability to
С. Antibiotics in this case? distinguish red and green colors. Which antituberculous
D. Antiseptics A. *Hydrazids of isonicotinic acid agent can cause this side effect?
E. Keratoplasty B. Barbiturates A *Ethambutol
С. Antibiotics-tetracyc/ines B. Streptomycin
8. Specify the agent which could be used for the D. β-adrenoblockers С. PAS
prevention of influenza during epidemic period. E. Nitrates D. Rifampicine
A. *Remantadinum
13.Which antituberculous drug from the following E. Cycloserine
B. Biseptolum
С. Ampicillinum oppresses transcription DNA to RNA? 18. Determine drug for AIDS treatment with following
A. *Rifampicine
44
mechanism of action: it is able to be phosphorilizated in B. Isoniazid E Ethoxide
cells and transformed to triphosphate, and then it inhibits C. Ethionamide
viral D. Streptomycin sulfas
transcriptase and impede of DNA synthesis from viral E. Ethambutol ANTIPROTOZOAL AND ANTHELMINTIC
RNA. 23. 39-years-old patient with pulmonary tuberculosis AGENTS.
A. *Zidovudine received effective complex treatment composed of 3
B. Saquinavir antituberculous agents including Steptomycin sulfate. What 1. A female patient addressed to gynecologist with
С. Indinavir is its mechanism of action? complaints of undant discharges from vagina with pleasant
D. Ritonavir A. *Blockade of proteins synthesis smell. After bacteriological vestigation the diagnosis of
E. Virasept B. PABA anti-metabolite trichmo-•sis has been given. Specify the ug which should
C. Blockade of RNA synthesis be administered.
19. After treatment by the antiretroviral drug from the D. Inhibition of DNA replication A. * Metronidazolum (Trichopolum)
group of nucleosides within six month a patient with AIDS F. Inhibition of mycolic acids synthesis В. Sulfadimezinum
developed reducing amount of RBC, neutrophilic cells and C. Chingaminum
platelet cells in the blood. Indicate the agent that is able to 24. A patient with primary syphilis has allergy to
benzylpenicillin. What drug can be prescribed in the D. Chloridinum
cause these side effects? E. Monomycinum
case?
A. *Zidovudine
A. *Erythromycin 2. Mother addressed to the pediatrician with the child who
B. Sackvinovir
B. Amoxicillin complained of strong itch in the region around the anus,
С. Acilovir
С. Amoxiclav pain intensified at night. After investigation of feaces the
D. Valaciclovir
D. Carbenicillin diagnosis of enterobiasis was given. Indicate the drug
E. Remantadine
E. Lincomycin which should be administered.
20. 19-Years old patient with primary syphilis receives 30. 25-years old woman was admitted to a hospital with A. *Levamisolum
complex treatment by benzylpenicillin natrium. What is its diagnosed syphilis. Indicate the main antibiotic for her B. Trichlorophenum
mechanism of action? treatment: С. Phenasalum
A. *Blockade of murein synthesis in cell walls D. Ditrazinum
A. *Benzylpenicillin natrium (penicillin G sodium)
B. Blockade of protein synthesis in cytoplasm E. Aminoacrichinum
B. Tetracycline
С. Blockade of tiotic groups of enzymes С. Lincomycin 3. Indicate the antimalarial agent which is active against
D. Blockade of RNA synthesis
D. Vancomycin paraerythrocytic forms of Plasmodium.
E. Blockade of DNA synthesis E. Erythromycin А *Рrimachinum
21. A patient has been suffering from tuberculosis 31. A patient suffers from pulmonary tuberculosis. During В Chingaminum
associated with intracellular location of mycobacterium treatment neuritis of visual nerve arose. What drug has С. Galochinum
for a long period of time. What drug must be included in caused this by-effect? D. Hydroxychlorochinum
complex treatment? A *Isoniazid E. Aminoacrichinum
A. Rifampicine B Ethambutol 4. Specify the drug which is used in amebiasis of any
B. *Isoniazid C Kanamycin localization of pathological process.
C. Ethambutol D Rifampicin A. *Metronidazolum (Trichopolum)
D. Ethionamide E Streptomycin B. Chingaminum
E. Natrium paraaminosalicilate 32. A patient was diagnosed with active focal pulmonary C. Emetinum hydrochloridum
tuberculosis. What drug should be prescribed in the first D. Chiniophonum
22. A patient known to be treated of pulmonary place?
tuberculosis noticed that his lachrymal liquid and urine E. Tetracyciinum
A *Isoniazid
became red. What drug is able to develop such side 5. A patient visited a physician with complaints of bowel
B Sulfalen
effect? dysfunction. After laboratory examination the diagnosis of
C Cyclocerine
A. *Rifampicine lambliasis was made. Specify the drug that should be used.
D Ethionamide
45
A. *Metronidazolum (Trichopolum) used as immunological modulator. Indicate the drug! 15. A patient consulted a doctor about bowels disfunction.
B. Tetracycilinum A. *Levamisole The doctor established symptoms of duodenitis and
С. Trichomonacid B. Piperazine enteritis. Laboratory examination helped to make the
D. Monomycinun С. Pyrantel following diagnosis: lambliosis. What medication should
E. Chingaminum D. Phenasaium (niclosamide) be administered?
6. During summer vacations a student from tropical E. Praziquantel A *Metronidazole
country developed tertian malaria. After recovery he B Erythromycin
11. A patient complaints of nausea, vomiting, loss of
turned back to Ukraine for study extension. In January an C Monomycin
appetite. After investigation of stool ascaridosis was
exacerbation was developed, it is known from past history D Chingamin
revealed. A drug with immune modulation activity was
of disease that drug acting on paraerythrocytic plasmodium E Tetracycline
prescribed for single usage. Indicate the drug:
malariae for prevention of relapse was not prescribed. 16. A patient ill with amebiasis was prescribed a certain
A. *Levamisole
Indicate the drug: drug. The use of alcohol together with this drug is contra-
B. Mebendazole
A. Chingaminum indicated because the drug inhibits metabolism of ethyl
С. Pyrantel
B. Halochin alcohol. What
D. Naphtamonum
С. Hydroxychloroquine drug is it?
E. Piperazine
D. Amodiaquine A *Metronidazole
12. A patient consulted a stomatologist about purulent
E. *Primaquine B Reserpine
inflammation of his gums. What drug will be the most
C Clonidine
7. Patient addressed to a physician to get a drug for effective if it is suspected that a causative agent is an
D Diazepam
prevention of malaria. Indicate the drug: anaerobe?
E Aminazine
A. *Primaquine A *Metronidazole
B. Clotrimazole B Gentamicin
С. Mebendazole C Oxacillin sodium
ANTITUMORAL AGENTS
D. Furazolidone D Co-trimoxazole
E. Fenasal E Nitroxoline
13. A 30-year-old patient complains about having 1. The antitumoral agent from the group of
8. What is the mechanism of anthelmintic action of abdominal pain and diarrhea for five days; body antimetabolites (antagonists of folic acid) was
levamisole? temperature rise up to $37,5^oC$ along with chills. The administered to the patient with acute leucosis.
A. *Oppression ot succinate dehydrogenase, day before a patient had been in a forest and drunk from an Indicate this drug.
ATPase open water reservoir. Laboratory analyses enabled to make A. *Methotrexatum
В. Oppression of MAO the following diagnosis: amebic dysentery. What is the B. Fluorouracil
C. DNA synthesis damage drug of choice for its treatment? C. Myelosanum
D. Cholinesterase activation A *Metronidazole D. Mercaptopurine
E. Oppression of N-acetyltransferase B Furazolidonum E. Hexestrolum
C Levomycetin 2. Determine a drug for treatment of lympholeukosis:
9. A drug is administered for prevention and treatment of
D Phthalazol
malaria, treatment of amebiasis and diseases of connective A. *Embichinum
E Emetine hydrochloride
tissue. Indicate the drug. B. Phthoruracilum
14. Systemic amebiasis with involvment of intestines,
A. *Chingaminum (chloroquine) C. Depostat
liver, lungs was diagnosed in a 52-year-old patient. What
B. Tetracycline D. Diethylstilbestrol
drug should be prescribed?
C. Metronidazole E. Phenobolinum
A *Metronidasol
D. Erythromycinum 3. A drug belongs to the group of anti-metabolites being
B Quiniofone
E. Quinine an antagonis causes impairment of purines' synthesis, and
C Tetracycline
10. A drug was administered to a patient with ascaridosis. D Quingamine thus lead to diminishing of nucleic acids' synthesis.
It is known to have influence on immune system, and is E Enteroseptol Determine the drug:
46
A. *Methotrexate E. Methylen blue B Isonitrozin
B. Mercaptopurine C Naloxone
4. A patient after drinking unknown liquid was admitted
C. Phthoruracilum (fluorouracil) D Activated carbon
to a hospital with complaints of stomachache, pain in
D. Cytarabine E Enterosorbent
gullet and diarrhea with little amount of blood. Under
E. Cispiatine 9. A patient suffering from syphilis has been treated with
examination hyperemia of throat mucous membranes,
bismuth preparations. As a result of it some grey spots
4. Antitumoral drug from the group of antimetabolites is gums' bleeding, lymphatic nodes enlargement and metallic
turned up on the mucous membrane of the oral cavity;
used for treatment of leucosis of children and cancer of aftertaste were revealed. Within 2-3 days renal
nephropathy symptoms were also present. What drug
adults. Determine the drug: insufficiency was developed. Determine the agent which
should be used for treatment of bismuth intoxication?
A. *Methotrexate caused named above symptoms:
A *Unithiol
B. Sarcolysinum A. *Salts of heavy metals B Methylene blue
С. Colchamine B. Furacilinum C Naloxone
D. Rubomycin С. Ethylic alcohol D Bemegride
E. Prednisolonum D. Boric acid E Nalorphine
E. Atropine
THE TREATMENT OF ACUTE POISONONGS BY
5. After treatment with bismuth preparation, a patient with GENERAL PHARMACOLOGY
DRUGS
syphilis developed symptoms of kidney insuffuciency and 1. A patient with chronic cardiac insufficiency has been
1. A 38 year-old patient was admitted to a hospital due to gray strains on mouth mucous membranes. Indicate a drug taking foxglove (Digitalis) preparations for a long time.
acute poisoning with mercury dichloride. Indicate the a doctor has to use as specific antidote in the case: Due to the violation of intake schedule the woman got
antidote which should be introduced to the patient A. *Unithiol symptoms of intoxication. These symptoms result from:
immediately. B. Bemegrid A *Material cumulation
A. * Unithiolum С. Nalorphine B Tachyphylaxis
B. Dipyroximum D. Naloxone C Idiosyncrasy
С. Atropinum E. Methylenum coeruleum (methylene blue) D Antagonism
D. Nalorphinum E Sensibilization
6. A 38 years-old male was admitted to ICU of a hospital 2. A man who has been taking a drug for a long time
E. Isonitrosinum
with severe poisoning by mercury chloride. Determine cannot withhold it because this causes impairment of
2. A patient was admitted to a hospital by the ambulance specific antidote: psychic, somatic and vegetative functions. Name the
due to acute poisoning with morphine. Indicate the agent A. *Unithiol syndrome of different disturbances caused by drug
which is used for gastric lavage in this situation. B. Dipiroxime discontinuation:
A. *Potassium permanganate С. Atropine A *Abstinence
B. Natrii hydrocarbonas D. Nalorphine B Sensibilization
С. Furacilinum E. Isonitrosine C Idiosyncrasy
D. Tanninum 7. A patient with acute morphine poisoning was delivered D Tachyphylaxis
E. Boric acid to a hospital. What specific narcotic antagonist should be E Cumulation
chosen in this case? 3. Proserin increases skeletal muscle tone when given
3. In a patient with syphilis there are grey stains on the
A *Naloxone systematically. Halothane induces relaxation of skeletal
oral cavity mucosa and the signs of nephropathy
B Paracetamol muscles and reduces proserin effects. What is the nature of
developed during treatment with bismuth compounds.
C Methacin proserin and halothane interaction?
Specify the agent which is used as antidote in poisonings
D Digoxin A *Indirect functional antagonism
by bismuth compounds,
E Unithiol B Direct functional antagonism
A. *Unithiolum
8. Patient with mercury poisoning was admitted to the C Competitive antagonism
В. Nalorphine
toxicological department from the chemical industry. What D Independent antagonism
С. Bemegridum
medicine should be used? E Noncompetitive antagonism
D. Naloxone
A *Unithiol 4. A patient with frequent attacks of stenocardia was
47
prescribed sustak-forte to be taken one tablet twice a prescribed an average therapeutic dose of digoxin. Two C. Preventive treatment
day. At first the effect was positive but on the second day weeks after begin of its taking there appeared symptoms of D. Symptomatic treatment
stenocardia attacks resumed. What can explain inefficiency drug intoxication (bradycardia, extrasystole, nausea). E. Pathogenic treatment
of the prescribed drug? Name the phenomenon that caused
A *Tachyphylaxis accumulation of the drug in the organism?
B Cumulation A *Material cumulation
C Sensibilization B Functional cumulation
D Idiosyncrasy C Tolerance
E Dependence D Tachyphylaxis
5. A patient taking clonidine for essential hypertension E Idiosyncrasy
treatment was using alcohol that caused intense inhibition 10. Mother of a 2 year old child consulted a stomatologist.
of central nervous system. What may it be connected with? In the period of pregnancy she was irregularly taking
A *Effect potentiating antibiotics for an infectious disease. Examination of the
B Effect summation child revealed incisor destruction, yellow enamel, brown
C Cumulation rim around the dental cervix. What drug has apparent
D Intoxication teratogenic effect?
E Idiosyncrasy A *Doxacycline
6. Continuous taking of some drugs foregoing the B Furosemide
pregnancy increase the risk of giving birth to a child with C Ampiox
genetic defects. What is this effect called? D Xantinol nicotinate
A *Mutagenic effect E Octadine
B Embryotoxic effect
11. During examination in out-patient department a
C Teratogenic effect
physician identified pneumonia and- prescribed in-patient
D Fetotoxical effect
treatment by ampicillin and cefalexin. However, the
E Blastomogenic effect
patient started treatment at home with the same antibiotics,
7. 36 y.o. man has a craniocerebral trauma. Objectively:
dosage and timing prescribed by the doctor. Within three
diminished breath sounds, thread pulse, no reflexes. What
days the sick person felt better, fever and cough reduced.
way of pyracetam introduction will be the most apropriate
The treatment was discontinued and the patient turned
in this case?
back to work. Next day he/she felt much worse, fever and
A *Intravenous
cough were developed again, that is why the patient had to
B Rectal
be examined by the physician, indicate please which
C Subcutaneous
principle of chemotherapy was disobeyed by the patient:
D Peroral
E Inhalation A. *Duration of treatment
8. A patient who has been suffering from cardiac B. Combined usage of agents
insufficiency for several months has been taking digoxin C. The earliest chemotherapy beginning
on an outpatient basis. At a certain stage of treatment there D. Effective agent choice based on clinical and
appeared symptoms of drug overdose. What phenomenon bacteriological diagnosis
underlies the development of this complication? E. Optimal selection of dosage, timing and
A *Material cumulation introduction ways of agent
B Habituation 12. Patient with pneumonia was treated by injections of
C Sensibilization antibiotic. Determine the type of chemotherapy:
D Functional cumulation A. *Causal treatment
E Tachyphylaxis B. Substitute treatment
9. A patient ill with chronic cardiac insufficiency was

You might also like